Dados Internacionais de Cataloga¸cao da Publicac¸ao(CIP) · 2018. 12. 13. · Resolu¸c˜ao...

64

Transcript of Dados Internacionais de Cataloga¸cao da Publicac¸ao(CIP) · 2018. 12. 13. · Resolu¸c˜ao...

Page 1: Dados Internacionais de Cataloga¸cao da Publicac¸ao(CIP) · 2018. 12. 13. · Resolu¸c˜ao Comentada das Provas da XXVI OMEG Valdivino Vargas Junior, Rogerio de Queiroz Chaves,
Page 2: Dados Internacionais de Cataloga¸cao da Publicac¸ao(CIP) · 2018. 12. 13. · Resolu¸c˜ao Comentada das Provas da XXVI OMEG Valdivino Vargas Junior, Rogerio de Queiroz Chaves,

Dados Internacionais de Catalogacao da Publicacao(CIP)(GPT/BC/UFG)

Revista da Olimpıada/Universidade Federal de Goias/Instituto de Matematica e EstatısticaNo- 13 (nov.2018/set.2019) Goiania: Editora da UFG, 2018-v. AnualMatematica - Periodicos - ISSN 1518-6075 - CDU: 51(05)

Comite EditorialFrancisco Bruno de Lima Holanda /FACE/UFG.Jose Hilario da Cruz /IME/UFG.Rogerio de Queiroz Chaves /IME/UFG.Ronaldo Alves Garcia /IME/UFG.Thaynara Arielly de Lima /IME/UFG.Ticianne Proenca Bueno Adorno /IME/UFG.

Editoracao Arte da CapaJose H. da Cruz Ana P. A. Chaves (capa)

Anderson V. Macedo (logomarca)

Tiragem Postagem440 exemplares 2o- semestre de 2018

Revista da Olimpıada, no- 13, 2018Universidade Federal de GoiasInstituto de Matematica e EstatısticaCampus Samambaia74.690-900 - Goiania - GoiasTel.: (62) 3521 1208Versao eletronica disponıvel em:

https://omeg.mat.ufg.br/p/399-revista-da-omeg

Artigos assinados sao da responsabilidade dos autores.E permitida a reproducao, desde que seja citada a fonte.

Page 3: Dados Internacionais de Cataloga¸cao da Publicac¸ao(CIP) · 2018. 12. 13. · Resolu¸c˜ao Comentada das Provas da XXVI OMEG Valdivino Vargas Junior, Rogerio de Queiroz Chaves,

Revista da Olimpıada - IME - UFG, no- 13, 2018

Apresentacao

Caro Leitor,

A Revista Olimpıada de Matematica do Estado de Goias e uma pu-blicacao anual do Instituto de Matematica e Estatıstica da UFG e temcomo principal publico alvo, professores e estudantes do ensino funda-mental e medio. Tem como meta ser um veıculo de: difusao cultural,integracao Universidade/Escola, espaco de criacao e reflexao crıtica so-bre a ciencia Matematica.

Esperamos que, na leitura dos artigos e problemas propostos e re-solvidos, o leitor faca anotacoes complementares, amplie seus conheci-mentos nas bibliografias citadas e principalmente, seja capaz de difundiroralmente e com naturalidade o conteudo assimilado transmitindo-o aseus colegas, amigos, pais, filhos, etc. Tambem gostarıamos de rece-ber sugestoes e problemas que serao submetidos a analise para possıvelpublicacao.

Acreditamos que o domınio da ciencia, em particular da matematica,e o seu bom uso sao fundamentais para o desenvolvimento da humani-dade e nossa atencao para este fato e que todos possam apreciar, aqui, ariqueza da matematica e sejam agentes transformadores para elevarmosa cultura matematica no nosso Estado e no nosso Paıs.

Goiania, novembro de 2018Os Editores.

Page 4: Dados Internacionais de Cataloga¸cao da Publicac¸ao(CIP) · 2018. 12. 13. · Resolu¸c˜ao Comentada das Provas da XXVI OMEG Valdivino Vargas Junior, Rogerio de Queiroz Chaves,

Universidade Federal de Goias

Edward Madureira BrasilReitor

Sandramara Matias ChavesVice-Reitor

Flavia Aparecida de OliveiraPro-Reitora de Graduacao - Prograd

Laerte Guimaraes Ferreira JuniorPro-Reitora de Pos-Graduacao - PRPG

Jesiel Freitas CarvalhoPro-Reitor de Pesquisa e Inovacao - PRPI

Lucilene Maria de SousaPro-Reitoria de Extensao e Cultura - Proec

Robson Maia GeraldinePro-Reitor de Administracao e Financas - Proad

Everton Wirbitzki da SilveiraPro-Reitoria de Gestao de Pessoas - Pro-Pessoas

Maısa Miralva da SilvaPro-Reitoria de Assuntos Estudantis - Prae

Maurıcio Donizetti PieterzackDiretor do Instituto de Matematica e Estatıstica

Olimpıada de Matematica do Estado de Goias

Comissao Organizadora da XXVI OMEG:Maurıcio Donizetti PieterzackOle Peter SmithRogerio de Queiroz ChavesRosane Gomes PereiraThaynara Arielly de Lima (Coordenadora)Ticianne Proenca Bueno AdornoValdivino Vargas Junior.

Universidade Federal de Goias - Instituto de Matematica e Estatıstica

Campus Samambaia - CEP 74.690-900 - Goiania-GO

E-mail: [email protected] Tel:(62)3521-1208 Fax:(62)3521-1180

Site: https://omeg.mat.ufg.br

Page 5: Dados Internacionais de Cataloga¸cao da Publicac¸ao(CIP) · 2018. 12. 13. · Resolu¸c˜ao Comentada das Provas da XXVI OMEG Valdivino Vargas Junior, Rogerio de Queiroz Chaves,

Revista da Olimpıada de Matematica do Estado de Goias i

Indice

Classificados na XXVI OMEG - 2017 1

Resolucao Comentada das Provas da XXVI OMEG 4

Consideracoes sobre o ensino de Matematica 24

Jogos e Brincadeiras 33

Potencia de um Ponto e Eixo Radical 41

Page 6: Dados Internacionais de Cataloga¸cao da Publicac¸ao(CIP) · 2018. 12. 13. · Resolu¸c˜ao Comentada das Provas da XXVI OMEG Valdivino Vargas Junior, Rogerio de Queiroz Chaves,

Revista da Olimpıada - IME - UFG, no- 13, novembro de 2018. 1-3

Classificados na XXVI OMEG - 2017

Nıvel 1 (6o- e 7o- anos do Ensino Fundamental)

Medalha de Ouro

⋆ Geovanna Bispo Almeida/Instituto Presbiteriano de Educacao -Goiania

⋆ Nathalya Cirqueira Moura/Colegio da Polıcia Militar de Goias JoseCarrilho - Goianesia

Medalha de Prata

⋆ Bruno de Moraes Dumont/Colegio WRJ - Goiania

⋆ Pedro de Oliveira Freitas/Colegio Marista - Goiania

⋆ Pedro Porto de Carvalho Nunes/Colegio Marista - Goiania

Medalha de Bronze

⋆ Aline Aurora Saurama da Silva/Colegio Studium - Goiania

⋆ Gustavo Galvao e Silva/Educandario Vila Boa - Goiania

⋆ Larissa Lemos /Colegio Delta Jardim Goias - Goiania

Mencao Honrosa

⋆ Ana Clara Franca Bueno/Colegio Santo Agostinho - Goiania

⋆ Enzo Antonio Reis Kivlan/Colegio Integrado Jao - Goiania

⋆ Igor Dias Aguiar/Escola Interamerica - Goiania

⋆ Isabella Goncalves/Colegio Delta Jardim Goias - Goiania

⋆ Luana Fialho Franco de Melo/Colegio Integrado Jao - Goiania

Page 7: Dados Internacionais de Cataloga¸cao da Publicac¸ao(CIP) · 2018. 12. 13. · Resolu¸c˜ao Comentada das Provas da XXVI OMEG Valdivino Vargas Junior, Rogerio de Queiroz Chaves,

Revista da Olimpıada de Matematica do Estado de Goias 2

⋆ Roberto Spındola Abrenhosa Filho/Colegio Crescer - Anapolis

⋆ Rodrigo Brom Gomes dos Santos/Educandario Vila Boa - Goiania

⋆ Tarcısio Oliveira Quintino/Colegio Nossa Senhora do Bom Conse-lho - Jataı

Nıvel 2 (8o- e 9o- anos do Ensino Fundamental)

Medalha de Ouro

⋆ Felipe Reis Spirandelli/Colegio WRJ - Goiania

⋆ Joao Victor Borges Guimaraes/Colegio WRJ - Goiania

Medalha de Prata

⋆ Ana Luıza Felix de Souza /Escola Interamerica - Goiania

⋆ Fernando G. Campos/Colegio Galileu - Anapolis

⋆ Gabriel Inumaru Esteves/Colegio Integrado Jao - Goiania

⋆ Izadora Caiado Oliveira/Colegio WRJ - Goiania

⋆ Jonatan de Lima Santos/Escola Municipal Pedro Gomes de Me-nezes - Goiania

Medalha de Bronze

⋆ Fernando Hiroshi Shimoyama/Colegio WRJ - Goiania

⋆Gustavo Henrique de Oliveira Carmo Borges/Colegio WRJ - Goiania

⋆ Gustavo Kerdole Gontijo/Colegio Progressivo - Goiania

⋆ Rui Andrade Carvalho Nunes/Colegio WRJ - Goiania

Mencao Honrosa

⋆ Ana Clara Schaitl Thon/Colegio Integrado Jao - Goiania

⋆ Arthur Fernandes de Melo/Colegio Santo Agostinho - Goiania

⋆ Camila Campos de Oliveira/Colegio Integrado Jao - Goiania

⋆ Gabriel Vieira de Figueiredo/Centro Educacional OMNI - Goiania

Page 8: Dados Internacionais de Cataloga¸cao da Publicac¸ao(CIP) · 2018. 12. 13. · Resolu¸c˜ao Comentada das Provas da XXVI OMEG Valdivino Vargas Junior, Rogerio de Queiroz Chaves,

Revista da Olimpıada de Matematica do Estado de Goias 3

⋆ Isabella Machado Doutor/Instituto Presbiteriano de Educacao -Goiania

⋆ Luiz Henrique Marques Goncalves/Colegio Olimpo - Goiania

Nıvel 3 (Ensino Medio)

Medalha de Ouro

⋆ Luiz Vasconcelos Junior/Colegio Estadual Americo Antunes - SaoLuıs de Montes Belos

⋆ Matheus Laureano Nunes Barbosa/Colegio Visao - Goiania

⋆ Vinıcius Alcantara Nevoa/Colegio Visao - Goiania

Medalha de Prata

⋆ Andre Felipe de Oliveira Fonseca/Colegio Integrado Jao - Goiania

⋆ Carlos Marcio Filho/Colegio Visao - Goiania

⋆ Luıs Henrique Zuin Ruiz/Colegio Olimpo - Goiania

Medalha de Bronze

⋆ Carlos Henrique Lopes Vitoriano/Colegio Olimpo - Goiania

⋆ Daniel Vandre Barbosa/Colegio Visao - Goiania

⋆ Gabriel Rezende Abrahao Pereira/Colegio Prepara Enem - Goiania

⋆ Lucas Oliveira Zago/Colegio Simbios - Goiania

⋆ Luiz Otavio da Matta Ambrosio/Colegio Agostiniano Nossa Se-nhora de Fatima - Goiania

⋆ Rafael Teles Borges Sobrosa/SESC Cidadania - Goiania

Mencao Honrosa

⋆ Gabriel Rodrigues Carvalho Paranhos/Colegio Delta Jardim Goias- Goiania

⋆ Marcio Roberto Rodrigues Filho/Colegio Prevest Centro - Goiania

⋆ Matheus de Paula Castro/Colegio Metropolitano - Goiania

⋆ Pedro Paulo Carvalho Vieira/SESC Cidadania - Goiania

⋆ Rafael Costa Ribeiro/Colegio Integrado Jao - Goiania

Page 9: Dados Internacionais de Cataloga¸cao da Publicac¸ao(CIP) · 2018. 12. 13. · Resolu¸c˜ao Comentada das Provas da XXVI OMEG Valdivino Vargas Junior, Rogerio de Queiroz Chaves,

Revista da Olimpıada - IME - UFG, no- 13, novembro de 2018. 4-23

Resolucao Comentada das Provas da XXVI

OMEG

Valdivino Vargas Junior, Rogerio de Queiroz Chaves,Rosane Gomes Pereira, Tiago Moreira Vargas,

Ana Paula Chaves, Francisco Bruno de Lima Holanda,Kamila da Silva Andrade, Abiel Costa Macedo

Resumo. Apresentamos uma resolucao comentada das questoes daOMEG de 2017. Incorporamos, na medida do possıvel, ideias e ar-gumentos apresentados por estudantes que participaram da olimpıada,fazendo mencao a seus nomes. Sugerimos que, antes de simplesmente leras solucoes, o leitor encare o desafio de resolver os problemas e desfruteda sensacao de conquista e de amadurecimento que esta atividade podeproporcionar. Por isso, todos os problemas sao apresentados primeiro e,separadamente ao final, as solucoes e comentarios.

Provas da XXVI OMEG

Nıvel 1

Problema 1: Um recipiente com capacidade para dois litros continha,inicialmente, um pouco de agua. Foram acrescentados, entao, outros200 mililitros de agua que ocuparam um quinto da capacidade que aindarestava do recipiente. Calcule a quantidade total de agua no recipienteapos esse acrescimo.

Problema 2: O idioma do planeta Ratom e o ratones. Observe astres frases em ratones, a seguir, e sua traducao para o portugues.

ρm my ty am = Gato gigante era assustador.

ty am λρ δy = Rato viu gato gigante.

ρm Γy ty λρ = Gato assustador comeu rato.

Page 10: Dados Internacionais de Cataloga¸cao da Publicac¸ao(CIP) · 2018. 12. 13. · Resolu¸c˜ao Comentada das Provas da XXVI OMEG Valdivino Vargas Junior, Rogerio de Queiroz Chaves,

Revista da Olimpıada de Matematica do Estado de Goias 5

Traduza para o ratones a frase “Rato gigante era gato.”

Problema 3: A figura a seguir representa um retangulo ABCD emque os pontos E, F , G e H sao os pontos medios dos lados.

(a) A area do triangulo JFC corresponde a qual fracao da area doretangulo IFCG?

(b) Se a area do triangulo CHE e de 120 cm2, qual e a area dotriangulo IJC?

Problema 4: Dois colegas, Alberto e Rafael, se encontraram em umonibus e comecaram a conversar. Uma parte do dialogo deles e transcritaa seguir.

A: “Quanto tempo! Da ultima vez que lhe vi voce havia acabado dese casar. E agora, ja tem filhos?”

R: “Sim, tenho 3 filhos.”

A: “Parabens! E quais sao as idades deles?”

R: “Bem, se voce multiplicar as idades deles obtera 36 mas se vocesoma-las obtera a quantidade de passageiros neste onibus. Alemdisso, eles tem idades distintas, o mais velho tem menos de 10anos e a diferenca de idades entre os dois mais novos e entre osdois mais velhos nao e maior que tres anos.”

Determine, a partir desse trecho do dialogo, as idades de cada uma dascriancas e a quantidade de passageiros no onibus.

Problema 5: Nos quadrados de um tabuleiro 3×3, sao escritos ossinais + e -, como nas figuras a seguir. Esses sinais podem ser trocados,

Page 11: Dados Internacionais de Cataloga¸cao da Publicac¸ao(CIP) · 2018. 12. 13. · Resolu¸c˜ao Comentada das Provas da XXVI OMEG Valdivino Vargas Junior, Rogerio de Queiroz Chaves,

Revista da Olimpıada de Matematica do Estado de Goias 6

desde que se troquem, de cada vez, todos os sinais de uma mesma linhaou todos de uma mesma coluna. Por exemplo, uma linha “++−” podeser trocada por “− − +”. Determine se e possıvel, ou nao, passar daconfiguracao do tabuleiro a esquerda, na figura, para a do tabuleiro adireita realizando uma sequencia desse tipo de trocas permitidas.

Problema 6: Uma sorveteria estava com uma promocao para algunssabores de sorvete, quando comprados em quantidades maiores: duasbolas de sorvete de morango por dez reais, tres bolas de sorvete de uvapor nove reais ou seis bolas de sorvete de limao por dois reais. Um grupode amigos decidiu comprar apenas sorvetes dessa promocao e comprouum total de 100 bolas contendo todos os tres sabores.

(a) Nessas condicoes, se o grupo nao gastou mais que 100 reais, quale a quantidade maxima de bolas de sorvete de morango que elespodem ter comprado?

(b) Se eles gastaram exatamente 100 reais, determine a quantidadeexata de bolas compradas de cada sabor.

Nıvel 2

Problema 1: O idioma do planeta Ratom e o ratones. Observe astres frases em ratones, a seguir, e sua traducao para o portugues.

ρm my ty am = Gato gigante era assustador.

ty am λρ δy = Rato viu gato gigante.

ρm Γy ty λρ = Gato assustador comeu rato.

Traduza para o ratones a frase “Rato gigante era gato.”

Problema 2: Uma caixa fechada, de vidro, em formato de para-lelepıpedo reto (todas as suas faces sao retangulos), esta parcialmente

Page 12: Dados Internacionais de Cataloga¸cao da Publicac¸ao(CIP) · 2018. 12. 13. · Resolu¸c˜ao Comentada das Provas da XXVI OMEG Valdivino Vargas Junior, Rogerio de Queiroz Chaves,

Revista da Olimpıada de Matematica do Estado de Goias 7

preenchida com um litro (1000 cm3) de agua. Quando a caixa e apoi-ada, de tres formas diferentes, sobre alguma de suas faces em uma mesahorizontal, a altura da parte ocupada pela agua e de 2 cm, 4 cm e 5 cm,respectivamente. Qual e a capacidade maxima, em litros, dessa caixa?

Problema 3: Determine dois numeros reais, x e y, tais que x3+ y3 =133

1000e x2y + xy2 =

7

100.

Problema 4: A figura a seguir representa um trapezio ABCD, comangulos retos em A e B. A parte triangular DEC corresponde a umaarea de 150m2 com o segmento AB medindo 20m e o segmento BCmedindo 10m.

(a) Deseja-se tracar um segmento EF que divida o trapezio em duaspartes de mesma area, como indica a figura a direita. Qual deveser a distancia F a C?

(b) Seja O o ponto medio do segmento EF descrito no item (a). Ima-gine que o ponto O seja mantido fixo e o segmento que vai de O aF seja girado em torno de O, como se fosse um pendulo, na direcaodo segmento AB. Nesse caso, a extremidade que inicialmente estaem F toca o segmento AB a que distancia de B?

Problema 5: Determine todos os numeros naturais x tais que oproduto dos seus dıgitos, em representacao decimal, e igual a x2−10x−22.

Problema 6: Um sapinho encontra-se em um plano cartesiano. Emqualquer ponto que esteja, com coordenadas dadas por (x, y), ele podeexecutar um dos seguintes movimentos:

• Pular do ponto (x, y) para o ponto (x+ 2, y + 4).

Page 13: Dados Internacionais de Cataloga¸cao da Publicac¸ao(CIP) · 2018. 12. 13. · Resolu¸c˜ao Comentada das Provas da XXVI OMEG Valdivino Vargas Junior, Rogerio de Queiroz Chaves,

Revista da Olimpıada de Matematica do Estado de Goias 8

• Pular do ponto (x, y) para o ponto (x, y + 5).

• Pular do ponto (x, y) para o ponto (x− 2, y − 9).

Inicialmente o sapinho encontra-se na origem do sistema de coordenadas,ou seja, no ponto (0, 0).

(a) Utilizando os movimentos permitidos, apresente uma sequencia depulos para que o sapinho va para a posicao (6, 7).

(b) Determine para quais valores de k e possıvel o sapinho chegar aoponto (k, 2017) utilizando uma sequencia dos movimentos permi-tidos.

Nıvel 3

Problema 1: Tres amigos, Andre, Bruno e Carlos jogam futebol comos alunos do curso de Matematica nas tardes de domingo. Sempre queAndre e Carlos vao jogar, Bruno tambem vai. Quando Andre nao vai,os amigos Bruno e Carlos tambem nao vao. Ate hoje, os alunos do cursode Matematica ja se reuniram 40 vezes para jogar. Desse total, Andrefoi jogar 30 vezes, algumas vezes Bruno nao foi e exatamente 19 vezesCarlos nao foi jogar. Com base nessas informacoes, determine os valoresmaximo e mınimo possıveis para a quantidade de vezes em que Brunonao foi jogar.

Problema 2: Potencias perfeitas sao numeros da forma ab, com a e binteiros e b > 1. Seja f(n) a maior potencia perfeita que nao excede n.Por exemplo, f(4) = 4, f(28) = 27 etc.

(a) Obtenha f(2017).

(b) Denote-se por fk(n) a composicao da funcao f consigo mesma kvezes, calculada em n. Por exemplo, f3(n) = f(f(f(n))). Calculef2017(2017!)

f(2017!)(Obs.: n! = n(n− 1)(n− 2) · · · 2 · 1 denota o fatorial

de n).

Problema 3: Considere um retangulo ABCD em que AB = 3 eBC = 1. Se P e um ponto qualquer do segmento CD, determine osvalores maximo e mınimo possıveis para o produto PA · PB.

Page 14: Dados Internacionais de Cataloga¸cao da Publicac¸ao(CIP) · 2018. 12. 13. · Resolu¸c˜ao Comentada das Provas da XXVI OMEG Valdivino Vargas Junior, Rogerio de Queiroz Chaves,

Revista da Olimpıada de Matematica do Estado de Goias 9

Problema 4: Mostre que, em qualquer conjunto de 7 numeros inteiros,positivos, distintos e menores que 127, e possıvel escolher dois deles,digamos a e b, tais que a < b ≤ 2a.

Problema 5: Uma urna possui inicialmente 40 bolas verdes e 10 bolasvermelhas. Dez bolas sao retiradas da urna, uma de cada vez, ao acasoe sem reposicao.

(a) Calcule a probabilidade de que a segunda bola retirada seja verde.

(b) Calcule a probabilidade de que a decima bola retirada seja verde.

(c) Sabendo-se que a terceira bola retirada foi verde, qual e a proba-bilidade de que a quarta retirada tambem seja verde?

Problema 6: Entre os habitantes de Saiog, quaisquer duas pessoasque nao se conhecem tem exatamente dois conhecidos em comum e duaspessoas que se conhecem nao tem outros conhecidos em comum. Mostreque, em consequencia disso, todos os habitantes tem, necessariamente,o mesmo numero de conhecidos.

Resolucao comentada

Nıvel 11. Um recipiente com capacidade para dois litros... (baseado na solucaoapresentada por Gabriel Yamin Nogueira)

Inicialmente, note que 2 l = 2000ml e que 200ml correspondem a1/10 de 2 l. Logo, o recipiente pode ser dividido em 10 partes iguais, comcapacidade de 200ml. Uma vez que 200ml corresponde a 1/5 da capa-cidade que restava no recipiente, concluımos que, inicialmente, havia 5partes vazias e, consequentemente, 5 preenchidas, o que corresponde a5×200ml = 1000ml. Portanto, a quantidade total de agua no recipienteapos esse acrescimo e 1200ml ou 1, 2 l.

2. O idioma do planeta Ratom e o ratones ... (baseado nas solucoesapresentadas por Aline Aurora Saurama da Silva, Ana Clara FrancaBueno e Ana Elisa Marques)

Page 15: Dados Internacionais de Cataloga¸cao da Publicac¸ao(CIP) · 2018. 12. 13. · Resolu¸c˜ao Comentada das Provas da XXVI OMEG Valdivino Vargas Junior, Rogerio de Queiroz Chaves,

Revista da Olimpıada de Matematica do Estado de Goias 10

A unica palavra que aparece nas tres frases em portugues e gato eem ratones e ty. Logo, ty = gato.

Em portugues, assustador e a unica palavra que aparece somente naprimeira e terceira frase. Em ratones, ρm. Logo, ρm = assustador.

Em portugues, gigante e a unica palavra que aparece somente naprimeira e segunda frase. Em ratones, am. Logo, am = gigante.

Em portugues, rato e a unica palavra que aparece somente na se-gunda e terceira frase. Em ratones, λρ. Logo, λρ = rato.

Em portugues, era e a unica palavra que aparece somente na pri-meira. Em ratones, my. Logo, my = era.

Em portugues, viu e a unica palavra que aparece somente na segundafrase. Em ratones, δy. Logo, δy = viu.

Em portugues, comeu e a unica palavra que aparece somente naterceira frase. Em ratones, Γy. Logo, Γy = comeu.

Entao, temos a seguinte tabela de correspondencia entre os idiomasportugues e ratones:

Portugues Ratones

gato tyassustador ρmgigante amrato λρera myviu δy

comeu Γy

Por fim, em todos os exemplos de frases em ratones dados, a traducaodas palavras para o portugues na ordem em que aparecem resulta naspalavras em ordem alfabetica em portugues. Se isso for sempre verdadeno ratones, a traducao da frase pedida e “my ty am λρ”.

Outra possibilidade e imitar a ordem das palavras para a frase doexemplo que mais se assemelha a frase pedida (”gato gigante era assus-tador”). Neste caso, a traducao ficaria “ty my λρ am”

3. A figura a seguir representa um retangulo ABCD em que os pontosE, F , G e H sao os pontos medios dos lados... (baseado na solucaoapresentada por Bruno de Moraes Dumont)

Page 16: Dados Internacionais de Cataloga¸cao da Publicac¸ao(CIP) · 2018. 12. 13. · Resolu¸c˜ao Comentada das Provas da XXVI OMEG Valdivino Vargas Junior, Rogerio de Queiroz Chaves,

Revista da Olimpıada de Matematica do Estado de Goias 11

(a) Iniciaremos a nossa solucao comparando os triangulos retangulosCJF e EJI. Comparando os angulos dos dois triangulos, conse-guimos concluir a semelhanca dos triangulos considerados inicial-mente. Alem disso, sendo E,F,G e H pontos medios dos ladostemos que: IE = FB = CF , isto e, a constante de proporcionali-dade da semelhanca de triangulos e igual a 1 e consequentemente,IJ = JF . Logo, J e ponto medio do segmento IF . Mais ainda,area(∆CJF ) = area(∆EJI).

A area do triangulo CIJ e dada por

area(∆CJI) =IJ · CF

2

Como IJ = JF entao

area(∆CJI) =JF · CF

2= area(∆CJF ). (1.1)

Seja A a area do retangulo IFCG. Observemos que

area(∆CIF ) =A

2(1.2)

Por outro lado,

area(∆CIF ) = area(∆CIJ) + area(∆CJF ).

Substituindo as informacoes obtidas em 1.1 e 1.2 na igualdadeacima concluımos que

area(∆CJF ) =A

4.

(b) Seja A a area do retangulo ABCD. Sendo E,F,G e H pontosmedios dos lados, temos que

A =A

4.

Por outro lado

Page 17: Dados Internacionais de Cataloga¸cao da Publicac¸ao(CIP) · 2018. 12. 13. · Resolu¸c˜ao Comentada das Provas da XXVI OMEG Valdivino Vargas Junior, Rogerio de Queiroz Chaves,

Revista da Olimpıada de Matematica do Estado de Goias 12

area(∆CHF ) = area(∆CHI) + area(∆CIF ) (1.3)

Como area(∆CIF ) =A

2e area(∆CHF ) = A entao obtemos de

1.3

area(∆CHI) =A

2

Sem muita dificuldade obtemos que

area(∆CHE) = area(∆CHI) + area(∆CJI)+

area(∆HEI) + area(∆EJI)(1.4)

Sabemos que area(∆CIJ) =A

4= area(∆EJI), area(∆CHI) =

A

2e area(∆HEI) =

A

2. Alem disso, area(∆CHE) = 120 cm2.

Substituindo as informacoes acima em 1.4 concluımos que

120 =A

2+

A

4+

A

2+

A

4⇒ A = 80 cm2

Logo, area(∆CIJ) = 20 cm2.

4. Dois colegas, Alberto e Rafael, se encontraram em um onibus ecomecaram a conversar... (baseado nas solucoes apresentadas por Geo-vanna Bispo Almeida, Tarcısio Oliveira Quintino, Luana Fialho Francode Melo e Mariany Barros Goncalves)

Sejam i1 < i2 <3 as idades dos filhos de Rafael e n a quantidade depessoas no onibus. Com as informacoes dadas temos

i1 · i2 · i3 = 36,i1 + i2 + i3 = n,i3 − i2 ≤ 3,i2 − i1 ≤ 3,i3 < 10.

Page 18: Dados Internacionais de Cataloga¸cao da Publicac¸ao(CIP) · 2018. 12. 13. · Resolu¸c˜ao Comentada das Provas da XXVI OMEG Valdivino Vargas Junior, Rogerio de Queiroz Chaves,

Revista da Olimpıada de Matematica do Estado de Goias 13

Como as idades sao numeros inteiros e que i3 < 10, devemos decom-por o numero 36 como produto de 3 numeros menores que 10. Notandoque 36 = 22 · 32 tem-se as seguintes possibilidades

9 · 4 · 1,9 · 2 · 2,6 · 6 · 1,6 · 3 · 2,4 · 3 · 3.

Dentre estas possibilidades, a unica que satisfaz as condicoes reque-ridas e i1 = 2, i2 = 3 e i3 = 6.

Assim, as idades sao 2, 3 e 6 e a quantidade de passageiros no onibuse n = 2 + 3 + 6 = 11.

5. Nos quadrados de um tabuleiro 3 × 3, sao escritos os sinais + e−, como nas figuras a seguir... (baseado na solucao apresentada porNathalia Cisqueira Moura)

Note que o sinal da primeira casa da segunda linha e o mesmo nosdois tabuleiros, logo ele precisaria ter sido trocado um numero par devezes. Ja o sinal da segunda casa desta mesma linha muda de “+”para “–” e precisaria ter sido trocado um numero ımpar de vezes. Istoso pode ser obtido trocando-se o sinal da primeira coluna um numerodiferente de vezes do da segunda coluna, sendo um par e o outro ımpar,uma vez que as trocas de sinais de uma mesma linha nao alteram arelacao de paridade entre vizinhos na mesma linha. Mas se os sinais dasduas primeiras colunas forem trocados dessa forma (um numero par devezes em uma e ımpar na outra), os sinais das duas primeiras colunasna primeira e na terceira linhas tambem teriam que ser diferentes, o quenao e o caso. Logo e impossıvel passar da configuracao do tabuleiro daesquerda para o da direita com as operacoes permitidas.

6. Uma sorveteria estava com uma promocao para alguns sabores desorvete, quando comprados em quantidades maiores... (baseado nassolucoes apresentadas por Pedro de Oliveira Freitas e Rodrigo Brom Go-mes dos Santos)

(a) O numero de bolas nas promocoes de morango e limao e par e, se ogrupo comprou 100 bolas, entao o numero de trios de bolas de uva

Page 19: Dados Internacionais de Cataloga¸cao da Publicac¸ao(CIP) · 2018. 12. 13. · Resolu¸c˜ao Comentada das Provas da XXVI OMEG Valdivino Vargas Junior, Rogerio de Queiroz Chaves,

Revista da Olimpıada de Matematica do Estado de Goias 14

tem que ser par, ou seja, no mınimo dois trios de bolas de uva. Orestante (94 bolas) pode ser distribuıdo entre morango e limao paratotalizar 100 bolas e nao ultrapassar 100 reais. A quantidade debolas de limao tem que ser multiplo de 6, o que permite 15×6 = 90bolas de limao e 2× 2 de morango, ou 14× 6 = 84 de limao e 5× 2de morango etc. Nessa sequencia, cada vez que se trocam 6 bolasde limao por 6 de morango, gastam-se 28 reais a mais. Essa ultimaja custaria 18 reais das 6 bolas de uva, mais 28 reais das 14× 6 delimao, mais 50 reais das 5 × 2 de morango, totalizando 96 reais.Entao, o maximo que o grupo pode ter comprado e 5 pares debolas de morango (R$ 50) .

(b) Denotando-se por m o numero de pares de bolas de morango que ogrupo comprou, u o numero de trios de bolas de uva e ℓ o numerode conjuntos de 6 bolas de limao, tem-se, das condicoes dadas,

2m+ 3u+ 6ℓ = 100 bolas e 10m+ 9u+ 2ℓ = 100 reais.

Da primeira dessas duas equacoes, como ja visto, conclui-se que uprecisa ser par e tambem 3(u + 2ℓ) = 100 − 2m e multiplo de 3,logo multiplo de 6 e m nao pode ser 1, nem 3, nem 4. Logo, m = 2ou 5 (no item (a) ficou estabelecido que 1 ≤ m ≤ 5).

Como 9u+ 2ℓ = 100− 10m e 3(u+ 2ℓ) = 100− 2m, para m = 5,tem-se 9u + 2ℓ = 50 e 3(u + 2ℓ) = 90 ⇒ 9u + 18ℓ = 270, ouseja, 16ℓ = 220, o que e impossıvel. Logo, m = 2, 9u+ 2ℓ = 80 e3(u+ 2ℓ) = 96 ⇒ 9u+ 18ℓ = 288, ou seja, 16ℓ = 208 ⇒ ℓ = 13e 9u = 54 ⇒ u = 6.

Portanto, o grupo comprou 2×2 de morango, 6×3 de uva e 13×6de limao.

Nıvel 2

1. O idioma do planeta Ratom e o ratones...

Ver resolucao do problema 2 do nıvel 1.

2. Uma caixa fechada, de vidro, em formato de paralelepıpedo reto...(baseado na solucao apresentada por Jonatan de Lima Santos)

Page 20: Dados Internacionais de Cataloga¸cao da Publicac¸ao(CIP) · 2018. 12. 13. · Resolu¸c˜ao Comentada das Provas da XXVI OMEG Valdivino Vargas Junior, Rogerio de Queiroz Chaves,

Revista da Olimpıada de Matematica do Estado de Goias 15

Considere que a caixa possua lados de medida a, b e c centımetros.Observamos que ao colocarmos a caixa sobre as diferentes faces e levando-se em conta o conteudo de um litro (1000cm3) de agua, tem-se

2ab = 1000

4bc = 1000

5ca = 1000

=⇒

ab = 500

bc = 250

ca = 200

Assim, multiplicando as equacoes, obtem-se

(abc)2 = 500× 250 × 200 = 25000000.

Portanto,abc = 5000,

o que nos da 5 litros de capacidade total da caixa.3. Determine dois numeros reais, x e y... (baseado na solucao apresen-tada por Ana Luiza Felix de Souza)

Temos

xy(x+ y) =7

100.Por outro lado,

(x+ y)3 = x3 + 3x2y + 3xy2 + y3 = x3 + y3 + 3xy(x+ y)

=133

1000+ 3 · 7

100=

343

1000.

Assim, x+ y = 7/10, o que leva ao sistema

x+ y = 710

xy = 110

Substituindo a segunda equacao na primeira tem-se

x+1

10x=

7

10⇒ 10x2 − 7x+ 1 = 0 ⇒ x =

1

2ou x =

1

5.

Substituindo de volta na segunda equacao do sistema, conclui-se que osnumeros reais em questao sao 1/2 e 1/5.

4. A figura a seguir representa um trapezio ABCD... (baseado nassolucoes apresentadas por Joao Victor Borges Guimaraes e Izadora Cai-ado Oliveira)

Page 21: Dados Internacionais de Cataloga¸cao da Publicac¸ao(CIP) · 2018. 12. 13. · Resolu¸c˜ao Comentada das Provas da XXVI OMEG Valdivino Vargas Junior, Rogerio de Queiroz Chaves,

Revista da Olimpıada de Matematica do Estado de Goias 16

(a) Observemos que a area do trapezio AABCD e dada por

AABCD = AABCE +ADEC

= 200 + 150

= 350m2

Desejamos que a igualdade abaixo seja verdadeira

AABFE = AEFCD =350

2= 175m2

Observemos que AEFCD = AECF +ADEC . Logo,

AECF = 25m2.

Por outro lado,

AECF =EC · CF

2

25 =20 · CF

2CF = 2, 5m

(b) Seja r a reta passando pelo ponto O paralela ao segmento CB.A reta r intercepta os segmentos EC e AB nos pontos Q e P ,respectivamente.

Consideremos os triangulos retangulos EQO e ECF . Pela seme-lhanca de triangulos concluımos que

EQ =EC

2e OQ = 1, 25m

Sabemos que PQ = 10m. Alem disso, PO = 8, 75m.

Agora, seja F ′ o ponto em que a extremidade do segmento que egirado em torno de O toca o segmento AB. O triangulo formadoPOF ′ e um triangulo retangulo. Como OF ′ = OF e PO = 8, 75,obtemos

PF ′2= OF

2 − (8, 75)2

Page 22: Dados Internacionais de Cataloga¸cao da Publicac¸ao(CIP) · 2018. 12. 13. · Resolu¸c˜ao Comentada das Provas da XXVI OMEG Valdivino Vargas Junior, Rogerio de Queiroz Chaves,

Revista da Olimpıada de Matematica do Estado de Goias 17

Mas P e ponto medio de AB, de modo que PB = 10 = PF ′+F ′B,e obtemos

(10 − F ′B)2 = OF2 − (8, 75)2

⇒ 100 − 20F ′B + F ′B2

= OF2 − (8, 75)2

Por outro lado, do triangulo retangulo ECF , obtemos

EF2

= EC2+FC

2(2OF )2 = (20)2+(2, 5)2OF

2= 100+

6, 25

4

E, consequentemente,

F ′B2 − 20F ′B + (8, 75)2 − 6, 25

4= 0

Logo, F ′B admite dois possıveis valores F ′B = 15m e F ′B = 5m,sendo que este ultimo corresponde ao primeiro ponto em que aextremidade do segmento que gira no sentido horario toca AB,como descrito na questao.

5. Determine todos os numeros naturais x...

Seja x ∈ N escrito como x = d1d2d3...dk, na base 10. Sabemos queos dıgitos di ∈ 0, 1, 2, . . . , 9, para todo i = 1, 2, . . . , k. Sendo assim,denotando o produto dos dıgitos por P = d1 × d2 × d3 × · · · × dk, asseguintes desigualdades sao validas:

x = d1d2d3...dk ≥ P = d1 × d2 × · · · × dk .

Assim, temos x ≥ P = x2 − 10x − 22, o que nos da x2 − 11x − 22 ≤ 0,cujas solucoes naturais sao 1, 2, . . . , 12. Como o unico destes numerosque satisfaz P = x2 − 10x− 22 e x = 12, esta e a unica solucao.

6. Um sapinho encontra-se em um plano cartesiano...(baseado na solucaoapresentada por Rui Andrade Carvalho Nunes)

(a) Observando-se que so o primeiro tipo de pulo do sapinho e queaumenta o valor da primeira coordenada, para ir de (0, 0) para(6, 7) e preciso realizar pelo menos tres pulos do primeiro tipo.

Page 23: Dados Internacionais de Cataloga¸cao da Publicac¸ao(CIP) · 2018. 12. 13. · Resolu¸c˜ao Comentada das Provas da XXVI OMEG Valdivino Vargas Junior, Rogerio de Queiroz Chaves,

Revista da Olimpıada de Matematica do Estado de Goias 18

Tres desses pulos levam o sapinho ao ponto (6, 12), de maneiraque e preciso diminuir o valor da segunda coordenada, o que sopode ser feito com o terceiro tipo de pulo, que tambem diminuiem dois o valor da primeira coordenada, levando ao ponto (4, 3).Basta, entao, dar mais um pulo do primeiro tipo para chegar aoponto (6, 7). Uma sequencia de movimentos que leva o sapinho de(0, 0) a (6, 7) e,

(0, 0) → (2, 4) → (4, 8) → (6, 12) → (4, 3) → (6, 7).

Mas a ordem dos tipos de pulos nao importa, entao ha outrassolucoes possıveis, com esse mesmo numero de movimentos, e in-finitas outras com mais pulos (uma vez que a combinacao dos trestipos de pulos, uma vez cada, resulta em um deslocamento nulo).

(b) E facil verificar que a ordem em que os pulos sao realizados naoafeta o ponto final em que o sapinho chega, embora altere o cami-nho. Alem disso, os tres tipos de pulos, quando combinados emquantidades iguais, resultam em um deslocamento nulo, ou seja, osapinho retorna ao ponto de partida. Dessa forma, ao investigarquais combinacoes de pulos levam a pontos da forma (k, 2017),basta considerar combinacoes de dois dos tres pulos.

Por exemplo, combinando-se p pulos do primeiro tipo e q do se-gundo, deseja-se que (0 + 2p, 0 + 4p + 5q) = (k, 2017), o queleva a k = 2p e 4p + 5q = 2017. Para escrever 2017 como somade um multiplo de 4 com um multiplo de 5, este ultimo pre-cisa ser ımpar, logo terminado em 5 e 4p precisa terminar em2, o que leva a 4p = 12, 32, 52, . . . , 2012. Desta forma, tem-sep = 3, 8, 13, 18, . . . , 498, 503, que sao numeros da forma 5n+3, comn ∈ 0, 1, 2, . . . , 100. Como k = 2p, os pontos da forma (k, 2017)que podem ser atingidos combinando pulos dos dois primeiros ti-pos sao aqueles em que k = 10n+6, com n ∈ 0, 1, 2, . . . , 100, ouseja, k = 6, 16, 26, 36, . . . , 1006.

De modo semelhante, combinando-se p pulos do primeiro tipo e qdo terceiro, deseja-se que (0 + 2p − 2q, 0 + 4p − 9q) = (k, 2017),de modo que k = 2(p − q) e 4p − 9q = 2017. Como o interesse edescobrir os valores de k que sao viaveis, podemos reescrever essasegunda equacao como 4(p−q)−5q = 2017, ou seja, 2k−5q = 2017,

Page 24: Dados Internacionais de Cataloga¸cao da Publicac¸ao(CIP) · 2018. 12. 13. · Resolu¸c˜ao Comentada das Provas da XXVI OMEG Valdivino Vargas Junior, Rogerio de Queiroz Chaves,

Revista da Olimpıada de Matematica do Estado de Goias 19

lembrando que k deve ser par, por ser o dobro da diferenca entrep e q. Observando-se que 5q precisa ser ımpar, logo terminadoem 5, obtem-se que 2k precisa ser um multiplo de 4 terminadoem 2. Por outro lado, 2k = 2017 + 5q e, como q ≥ 1, obtem-se k ≥ 1011 e, entao, 2k = 2032, 2052, 2072, . . . . Ou seja, k =1016, 1026, 1036, . . . que e o mesmo que dizer que k = 10n + 6,com n ≥ 101, n ∈ N.

Por fim, combinando-se p pulos do segundo tipo e q do terceiro,deseja-se que (0−2q, 0+5p−9q) = (k, 2017), de modo que k = −2qe 5p− 9q = 2017. Aqui, se 5p for um numero terminado em 0, 9qtem que terminar em 3 e, lembrando que a soma dos dıgitos de ummultiplo de 9 e multiplo de 9, tem-se 9q = 63, 153, 243, 333, . . . ⇒9q = 63+ 90n ⇒ q = 7+ 10n, n ≥ 0, inteiro. Se 5p terminar em5, 9q tem que terminar em 8, ou seja 9q = 18, 108, 198, 288, · · · =18 + 90n ⇒ q = 2 + 10n, n ≥ 0, inteiro. Esses dois casos podemser combinados de forma que, para qualquer q = 2 + 5n, n ≥ 0,inteiro, e possıvel arranjar um p viavel tal que 5p − 9q = 2017 e,nesse caso, k = −2q = −4− 10n, que e o mesmo que 6+10n, comn < 0, inteiro.

Portanto, os valores de k tais que o sapinho consegue chegar aoponto (k, 2017) sao os da forma k = 6 + 10n, n ∈ Z.

Nıvel 3

1. Tres amigos, Andre, Bruno e Carlos, ... (baseado na solucaoapresentada por Carlos Henrique Lopes Vitoriano)

Como Andre foi jogar 30 vezes, entao ele nao foi jogar 10 vezes.Assim, Bruno nao foi jogar no mınimo 10 vezes, pois quando Andre naovai jogar Carlos e Bruno nao vao. Das 19 vezes que Carlos nao foi, 10sao exatamente as vezes em que Andre nao foi, sobrando 9 outras faltasde Andre e 21 dias em que os tres, necessariamente foram, ja que sempreque Andre e Carlos vao jogar, Bruno tambem vai. Conclusao: Brunonao foi jogar no mınimo 10 vezes e no maximo 19 vezes.

2. Potencias perfeitas sao numeros da forma... (baseado na solucaoapresentada por Luiz Vasconcelos Junior)

Temos que f(n) = maxab | a, b ∈ Z, b > 1 e ab ≤ n.

Page 25: Dados Internacionais de Cataloga¸cao da Publicac¸ao(CIP) · 2018. 12. 13. · Resolu¸c˜ao Comentada das Provas da XXVI OMEG Valdivino Vargas Junior, Rogerio de Queiroz Chaves,

Revista da Olimpıada de Matematica do Estado de Goias 20

(a) Para n = 2017 vamos buscar o maior numero da forma:

• a2 que seja menor ou igual a 2017: note que, 442 = 1936 e452 = 2025, logo se 0 < m < 44 entao m2 < 442 ≤ 2017 e seq > 44 tem-se 2017 < q2.

• 2b que seja menor ou igual a 2017: neste caso, 210 = 1024 e211 = 2048, logo se m > 2 entao m11 > 2017.

Com estes dados basta procurarmos um numero da forma ab com2 ≤ a ≤ 44 e 2 ≤ b ≤ 10. Vejamos:

• Para b = 3 temos: 123 = 1728 e 133 = 2197.

• Para b = 4 temos: 64 = (62)2 < 442 e 74 = (72)2 > 452.

• Para b = 5 temos: 45 = 210 = 1024 e 55 = 3125.

• Para b = 6 temos: 36 = 729 e 46 = 212 > 211 > 2017.

• Para b = 7 temos: 37 = 2187.

Daı, para 7 < b < 10 e m > 2 tem-se m7 > 2017 e 2b < 210 = 1024.Logo, a potencia perfeita procurada esta entre as listadas acima,concluindo que f(2017) = 442.

(b) Note que f(ab) = ab entao, se f(n) = ab tem-se f2(n) = f(f(n)) =f(ab) = ab = f(n). Portanto, fk(n) = f(n) para qualquer k ∈ N.Segue que

fk(2017!)

f(2017!)=

f(2017!)

f(2017!)= 1.

3. Considere um retangulo ABCD... (baseado na solucao apresentadapor Luiz Vasconcelos Junior)

Independentemente da localizacao de P em CD, a base AB do trianguloPAB mede 3 e a altura e 1. Logo, sua area e 3/2. Por outro lado,considerando-se o angulo θ entre PA e PB, a area do triangulo PAB

Page 26: Dados Internacionais de Cataloga¸cao da Publicac¸ao(CIP) · 2018. 12. 13. · Resolu¸c˜ao Comentada das Provas da XXVI OMEG Valdivino Vargas Junior, Rogerio de Queiroz Chaves,

Revista da Olimpıada de Matematica do Estado de Goias 21

tambem e 12PA · PB · sen θ e, obviamente, 0 < θ < π, para P ∈ CD.

Assim, 0 < sen θ < 1 e

PA · PB =3

sen θ.

Por isso, quanto mais proximo de π/2 estiver θ, maior e o sen θ e menore o produto PA · PB. O oposto ocorre quando θ se afasta de π/2. Nopresente caso, observando-se os pontos de interseccao de um semicırculode diametro AB com o retangulo em questao, observa-se que ha, defato, pontos em que θ = π/2, em que se atınge o mınimo PA · PB = 3.Quanto aos pontos em que θ mais se afasta de π/2, esses podem ser osextremos C ou D ou o ponto medio de CD. Para os primeiros, como adiagonal do retangulo mede

√9 + 1, tem-se sen θ = 3/

√10, o que leva a

PA · PB =√10. Para P no ponto medio de CD, tem-se

sen θ = 2 senθ

2cos

θ

2= 2

(3/2)√

(

3

2

)2+ 1

· 1√

(

3

2

)2+ 1

=12

13.

Mas, mais simplesmente, tem-se

PA · PB = PA2

=

(

3

2

)2

+ 1 =13

4= 3, 25 >

√10,

uma vez que 3, 252 > 10.Portanto, o valor mınimo de PA · PB e 3 e o maximo e 3,25.

4. Mostre que, em qualquer conjunto de 7 numeros inteiros... (baseadona solucao apresentada por Victor Ferraz de Castro Ribeiro)

Consideremos sete inteiros positivos c1 < c2 < c3 < c4 < c5 < c6 <c7 < 127. Para que nao haja dois elementos, ci e cj tais que ci < cj ≤ 2ci,e necessario que ci+1 ≥ 2ci + 1, o que leva a cadeia de desigualdades

c7 ≥ 2c6 + 1 ≥ 4c5 + 3 ≥ 8c4 + 7 ≥ 16c3 + 15 ≥ 32c2 + 31 ≥ 64c1 + 63.

Mas c1 e, no mınimo, 1, o que implica que c7 teria que ser maior que ouigual a 64 · 1 + 63 = 127, o que contraria a hipotese inicial. Logo, entreos sete numeros ha, necessariamente, ci e cj tais que ci < cj ≤ 2ci.

5. Uma urna possui inicialmente 40 bolas verdes e 10 bolas vermelhas...

Sejam os eventos: Vi: sair bola verde na i-esima retirada, i = 1, 2, · · · , 10.

Page 27: Dados Internacionais de Cataloga¸cao da Publicac¸ao(CIP) · 2018. 12. 13. · Resolu¸c˜ao Comentada das Provas da XXVI OMEG Valdivino Vargas Junior, Rogerio de Queiroz Chaves,

Revista da Olimpıada de Matematica do Estado de Goias 22

(a) Usando o P.F.C. tem-se

|Ω| = A50,10 =50!

40!e |V2| = 40 · A49,9 = 40 · 49!

40!.

Assim,

P(V2) =|V2||Ω| =

40 · A49,9

A50,10=

40 · 49!

40!

50!40!

=4

5.

Outra solucao seria

P(V2) = P(V1)P(V2|V1) + P(V C1 )P(V2|V C

1 )

=

(

40

50

)(

39

49

)

+

(

10

50

)(

40

49

)

=1.960

2.450=

4

5.

(b) Usando o P.F.C. tem-se

|V10| = 40 · A49,9.

Assim,

P(V10) =|V10||Ω| =

40 ·A49,9

A50,10

=40 · 49!

40!

50!

40!

=4

5.

(c) Usando o P.F.C. tem-se

|V3| = 40 · A49,9 e |V3 ∩ V4| = 40 · 39 · A48,8.

Assim,

P(V4|V3) =|V3 ∩ V4|

|V3|=

40 · 39 · A48,8

40 ·A49,9=

40 · 3948!

40!

40 · 49!40!

=39

49.

6. Entre os habitantes de Saiog...

Sejam X e Y dois habitantes de SAIOG, e denote por x1, x2, . . . , xke y1, y2, . . . , yl os conhecidos de X e Y , respectivamente. Se X e Yse conhecem, entao x1, x2, . . . , xk − Y e y1, y2, . . . , yl − X saodisjuntos, pois caso contrario terıamos um conhecido em comum entreX e Y . Mais ainda, como entre os xi’s ninguem se conhece, pois casocontrario terıamos pessoas que se conhecem com mais de um conhecido

Page 28: Dados Internacionais de Cataloga¸cao da Publicac¸ao(CIP) · 2018. 12. 13. · Resolu¸c˜ao Comentada das Provas da XXVI OMEG Valdivino Vargas Junior, Rogerio de Queiroz Chaves,

Revista da Olimpıada de Matematica do Estado de Goias 23

em comum, e o mesmo vale para os yi’s, entao para cada xi, excluindoY , existe um unico yj , diferente de X, tais que xi e yj se conhecem. Istose da por Y e xi nao se conhecerem, donde devem ter dois conhecidoscomuns, onde um deles e X e o outro deve ser um dos conhecidos de Y ,ou seja um yj. Portanto, temos uma correspondencia biunıvoca entreestes dois conjuntos, donde k = l.

Caso X e Y nao se conhecam, eles tem dois conhecidos em comum,digamos que um deles e Z. Entao X conhece Z e Y conhece Z, daıda primeira parte segue que a quantidade de pessoas que X conhece eigual a quantidade de pessoas que Z conhece que por sua vez, e igual aquantidade de pessoas que Y conhece. Logo, a quantidade de pessoasque X e Y conhecem e a mesma.

Page 29: Dados Internacionais de Cataloga¸cao da Publicac¸ao(CIP) · 2018. 12. 13. · Resolu¸c˜ao Comentada das Provas da XXVI OMEG Valdivino Vargas Junior, Rogerio de Queiroz Chaves,

Revista da Olimpıada - IME - UFG, no- 13, novembro de 2018. 24-32

Consideracoes sobre o ensino de Matematica1

MANFREDO PERDIGAO DO CARMO2

O Ministerio da Educacao achou oportuno que um representantedo Instituto de Matematica Pura e Aplicada (I.M.P.A.) do ConselhoNacional de Pesquisas participasse desta reuniao sobre o ensino da Ma-tematica e expressasse o ponto de vista do matematico profissional sobreos assuntos aqui tratados.

O I.M.P.A. tem um interesse vital nestas questoes. Primeiro, porquea modernizacao de qualquer currıculo e essencialmente a transferencia deconquistas cientıficas ao nıvel do ensino. Ninguem melhor, portanto, doque o matematico familiarizado com estas conquistas para julgar do va-lor, utilidade e limites desta transferencia. Segundo, e isto parece para-doxal, porque o I.M.P.A. atua precipuamente na pos-graduacao em Ma-tematica. O paradoxo, entretanto, desaparece quando levamos em contaque o desenvolvimento de uma pos-graduacao em numeros e nıveis capa-zes de fornecer as universidades, industrias e consultorias o contingentenecessario ao desenvolvimento brasileiro repousa, em ultima analise, emum ensino basico objetivo e correto.

Parece-nos claro que a elaboracao dos currıculos de Matematica paraa escola secundaria e uma tarefa conjunta de professores e matematicosativos em pesquisas. De fato, a nao ser que se queira ensinar uma outracoisa, e nao a Matematica tal como ela e entendida atualmente pelos ma-tematicos, a participacao destes e indispensavel. Cabe-lhes, por sua ex-periencia, indicar pontos que se tornaram obsoletos, sugerir renovacoes

1*Recebido pela SBM em 14 de novembro de 1973.2**o autor deseja agradecer a Leny Cavalcante, Elon Lima e Maurıcio Peixoto,

que leram o original e apresentaram valiosas sugestoes.**Conferencia apresentada na Reuniao sobre o Ensino da Matematica patrocinado

pelo Departamento de Ensino Fundamental do M.E.C., em Outubro de 1973.*** Publicado originalmente no Boletim da SBM, vol. 5, no.1, (1974) pp. 105-112.

Page 30: Dados Internacionais de Cataloga¸cao da Publicac¸ao(CIP) · 2018. 12. 13. · Resolu¸c˜ao Comentada das Provas da XXVI OMEG Valdivino Vargas Junior, Rogerio de Queiroz Chaves,

Revista da Olimpıada de Matematica do Estado de Goias 25

que se fazem necessarias, e determinar objetivos a serem atingidos alongo prazo. A Matematica brasileira ja atingiu uma maturidade sufici-ente para determinar, de maneira independente, os caminhos que maisnos convem.

E necessario salientar aqui a indispensavel participacao do professorna elaboracao destes projetos. Sua experiencia indicara as modificacoesnecessarias a transforma-los em realidade, levando em conta as condicoesbrasileiras.

A elaboracao de currıculos e, portanto, uma tarefa a longo prazo. Eladeve ser precedida, entretanto, de consideracoes gerais sobre o ensino daMatematica que, a nosso ver, se aplicam a todos os graus. E neste nıvelque se situa a nossa colaboracao ao presente debate.

Provavelmente, a razao fundamental para a introducao da Matema-tica no ensino basico, repousa no fato de que ela fornece instrumentosefetivos para compreender e atuar no mundo que nos cerca. Esta ati-tude de resolver problemas propostos pelo mundo real esta na propriabase da criacao da Matematica e tem sido uma fonte de inspiracao erenovacao de seus metodos. Talvez convenha mencionar que a palavraproblema sera utilizada aqui em um sentido lato, que tanto pode signi-ficar obter um metodo que permita determinar a distancia da Terra aLua, como tentar provar que a relacao entre o comprimento da circun-ferencia e do seu diametro nao pode ser expressa como o quociente dedois inteiros. Pode se dizer, sem exagero, que as estruturas mais abstra-tas em Matematica foram criadas para resolver problemas e que o graude validez e permanencia de uma estrutura mede-se pela multiplicidadede problemas que ela permite atacar. Estes problemas, convem repetir,nao sao necessariamente aplicaveis mas tem suas origens e motivacao emquestoes propostas pelo mundo real, e suas solucoes refletem uma me-lhor compreensao deste mundo. Esta e a Matematica de Arquimedes,Gauss, Riemann e Poincare e da qual queremos e devemos transmitiruma parte aos nossos alunos.

Das razoes acima, que pertencem a propria natureza da Matematica,decorre que a melhor maneira de ensina-la e atraves de problemas.Os conceitos a serem introduzidos devem ser aqueles que sejam indis-pensaveis a compreensao e solucao dos problemas propostos e que ilu-minem suas relacoes com outros problemas e conceitos ja estudados. A

Page 31: Dados Internacionais de Cataloga¸cao da Publicac¸ao(CIP) · 2018. 12. 13. · Resolu¸c˜ao Comentada das Provas da XXVI OMEG Valdivino Vargas Junior, Rogerio de Queiroz Chaves,

Revista da Olimpıada de Matematica do Estado de Goias 26

compreensao clara dos fatos e a habilidade de bem utiliza-los deve terpreferencia sobre a formalizacao. Quando for conveniente e possıvel, aformalizacao deve ser utilizada, porem sempre de maneira local. Porformalizacao local entendemos o seguinte. Dado um determinado as-sunto, admitimos certas premissas, que podem ter sido demonstradasem outros assuntos ou constituem elementos intuitivos bastante obviospara que se possa trabalhar com eles. A partir destas premissas, todasas definicoes sao explicitadas e todas as proposicoes sao demonstradas.

Um dos maiores mal-entendidos do ensino da Matematica proveioda adocao dos livros de Euclides, ou de pequenas modificacoes deles, noensino de Geometria. De inıcio, demos absolver Euclides de toda e qual-quer culpa no caso. Euclides escreveu os seus livros com uma finalidademetodologica e nao didatica. A formalizacao global, por ele obtida, dovolume de fato geometricos conhecidos ate entao foi uma obra de genio,melhor compreendida por filosofos e pensadores do que por jovens es-tudantes. Em oposicao a Arquimedes, que usava uma combinacao deformalizacao local e metodos heurısticos e cujas tecnicas de pesquisascontinham o germe de uma forma de ensino mais efetiva, a obra deEuclides foi tomada como um modelo didatico. As consequencias desas-trosas deste fato se fazem sentir ate hoje.

Queremos deixar bem claro que a contribuicao metodologica de Eu-clides e enorme. Pela primeira vez mostrava-se a possibilidade de umaformalizacao global que, embora com defeitos, levantava a esperanca dese estender a ideia de formalizacao a toda a Matematica e, quem sabe, apropria Ciencia. Hoje parece cada vez mais claro que a formalizacao ab-soluta da Matematica e um ideal inatingıvel que deve ser abandonado.Seja como for, formalizacoes locais relativamente amplas tiveram umsucesso consideravel na Mecanica em certos ramos de Fısica Teorica,na Algebra, etc, e a inspiracao para isto, se deve indiscutivelmente aEuclides.

Do ponto de vista didatico, porem, o problema fundamental e que aapresentacao formal da Matematica, na maior parte dos casos, escondee dissimula os mecanismos de criacao. Somente os alunos muito bemdotados sao capazes de apreciar este tipo de apresentacao, e estes setransformarao em matematicos sem que precisemos nos preocupar comeles. O aluno medio, com pouca experiencia em Matematica (e este, e o

Page 32: Dados Internacionais de Cataloga¸cao da Publicac¸ao(CIP) · 2018. 12. 13. · Resolu¸c˜ao Comentada das Provas da XXVI OMEG Valdivino Vargas Junior, Rogerio de Queiroz Chaves,

Revista da Olimpıada de Matematica do Estado de Goias 27

caso que nos interessa aqui), tem uma profunda dificuldade em acompa-nhar longas formalizacoes. A tendencia e reagir com desgosto e medo, ouaceita-las como um dogma, com grandes prejuızos para a sua imaginacaocriadora.

Mesmo para aqueles que vao se transformar em matematicos e anossa experiencia no I.M.P.A. e aı bem ilustrativa, o excesso de forma-lizacao pode facilmente se transformar em um freio ao processo criador.O objetivo da Matematica e, devemos repetir, resolver problemas. Sim-ples, no caso do ensino, complexos e em aberto, no caso de pesquisas.Em qualquer dos casos, a intuicao e a imaginacao criadora desempenhamum papel fundamental e a formalizacao e apenas um elemento auxiliar.Neste sentido, o espırito de Arquimedes e muito mais adequado a Ma-tematica atual do que o espırito de Euclides.

Convem neste momento chamar a atencao para uma falacia muitofrequente que consiste em confundir pensar corretamente com pensaraxiomaticamente. Em Matematica, como de resto em tudo mais, e ne-cessario pensar corretamente. Outra coisa nao fizeram matematicos ilus-tres do passado, tais como Gauss, Riemann e Poincare, que nos legaram oimenso edifıcio de que hoje dispomos, sem utilizar o metodo axiomatico.

Esta falacia e tanto mais grave por estar presente, de maneira im-plıcita, em uma certa atitude em relacao ao ensino da Matematica quese auto-denomina de Matematica Moderna. Deixando de lado a im-propriedade do termo, passarei a chama-los de “modernistas”, e creionao cometer injustica ao definir, segundo Papy, a Matematica Modernacomo o ensino da Matematica atraves das estruturas fundamentais. Es-truturas fundamentais foi um termo criado pela escola Bourbaki paradesignar certos conceitos logicamente simples (grupo, ordem, topologia)que aparecem reiteradamente na formacao de conceitos matematicos lo-gicamente complexos (se bem que intuitivamente mais simples). Se-gundo afirmacoes do psicologo Jean Piaget, as estruturas fundamentaiscorrespondem a certas categorias basicas do pensamento humano. Par-tindo destas premissas, os “modernistas” acreditam que a Matematicadeva ser ensinada a partir das estruturas fundamentais. A compreensaoexplıcita destas estruturas facilitaria enormemente o processo de apren-dizagem de todo o resto, que decorreria daı de uma maneira natural.Em resumo, um Bourbaki para criancas.

Page 33: Dados Internacionais de Cataloga¸cao da Publicac¸ao(CIP) · 2018. 12. 13. · Resolu¸c˜ao Comentada das Provas da XXVI OMEG Valdivino Vargas Junior, Rogerio de Queiroz Chaves,

Revista da Olimpıada de Matematica do Estado de Goias 28

Ora, como todos sabem, Bourbaki e um descendente direto de Eucli-des. No seu livro “Elementos de Matematica” publicado na decada de40, a escola Bourbaki tentou, segundo suas proprias palavras, “tomar aMatematica no seu inıcio” e reconstruı-la inteira a partir das estruturasfundamentais. Seria uma extensao de formalizacao de Euclides, com ascorrecoes necessarias, a toda a Matematica. Como e bem conhecido portodos os matematicos que trabalharam em pesquisas, o escopo global doprojeto falhou. As areas mais representativas da matematica atual, naopuderam ser encaixadas na filosofia do projeto inicial.

Do mesmo modo que no caso de Euclides, a influencia metodologicado projeto Bourbaki foi bastante grande. Entretanto, do ponto de vistada pesquisa, sua influencia foi praticamente inexistente. Em verdade, ovigoroso desenvolvimento da Matematica nas ultimas decadas foi feitodentro de um espırito que relega o formalismo a um segundo plano eignora completamente a existencia de Bourbaki. Deste ponto de vistapodemos afirmar que uma didatica da Matematica que insiste em seusaspectos formais estao em contradicao com o espırito da verdadeira Ma-tematica Moderna, isto e, aquela que se faz em nossos tempos. Uma taldidatica deve ser portanto considerada velha e obsoleta.

Admitida a verdade da afirmacao de Piaget, fato discutıvel mas cujaapreciacao nos desviaria do ponto principal desta exposicao, restam,entre outras as seguintes objecoes aos metodos “modernistas”:

1) Nao ha nenhuma razao de se esperar que o fato de explicitar verbal-mente os mecanismos basicos do pensamento ajude o seu desenvol-vimento operacional. Por exemplo, como observou Rene Thom, eduvidoso que o conhecimento previo das estruturas basicas de umalıngua ajude alguem a se exprimir com fluencia naquela lıngua.Pelo contrario, o que acontece com mais frequencia e que a tenta-tiva de instilar este conhecimento prematuro exerce um efeito defreagem no processo de aprendizagem. Este exemplo pode ser fa-cilmente multiplicado e mostra que o estudo extemporaneo das es-truturas fundamentais pode dificultar, em vez de facilitar, o apren-dizado da Matematica.

2) As estruturas fundamentais representam um longo processo de ela-boracao a partir de elementos que nos sao fornecidos pelo mundo

Page 34: Dados Internacionais de Cataloga¸cao da Publicac¸ao(CIP) · 2018. 12. 13. · Resolu¸c˜ao Comentada das Provas da XXVI OMEG Valdivino Vargas Junior, Rogerio de Queiroz Chaves,

Revista da Olimpıada de Matematica do Estado de Goias 29

real. Por serem logicamente mais simples, elas necessitam parao seu tratamento de uma maior formalizacao. Ora, o espırito deformalizacao e precisamente a antıtese do espırito da Matematicaatual e seria curioso que quisessemos modernizar os nossos currı-culos com ideias que comecam a se tornar obsoletas.

3) Como ja foi observado, os modernistas confundem rigor substancialcom rigor formal. Um exemplo esclarecera o que queremos dizer.Vamos supor que liguemos por uma reta dois pontos de um plano,um no interior e o outro no exterior de uma elipse neste plano.A afirmacao que a reta em questao encontra a elipse e substanci-almente rigorosa. E possıvel formalizar uma demonstracao destaafirmacao, porem no caso isto seria inutil. Dentro do contextode um trabalho de pesquisa, uma tal demonstracao seria um pe-dantismo. Dentro do contexto do ensino basico, a demonstracaoseria enfadonha e desviaria a atencao para pontos nao relevantesao assunto.

O rigor formal tem um certo papel a desempenhar em Matematicamas aplica-lo em situacoes como a acima revela uma falta de compre-ensao da natureza da Matematica. De fato, o rigor formal absoluto deuma teoria so podera ser atingido com a total ausencia de significadodos elementos da teoria. Pois uma tal teoria seria um conjunto de pro-posicoes da forma “p implica q”, onde nada sabemos sobre p ou q e nemsequer se q e verdade; apenas garantimos que q decorre de p segundoas regras de uma certa logica. Como disse Bertrand Russel, e uma teo-ria da qual nao sabemos do que estamos falando nem se o que estamosdizendo e verdade. Seria curioso especular sobre o nıvel de demenciaque teria se instalado em Matematica, tivesse ela seguido esta direcaode rigor formal absoluto. Entretanto, o rigor formal e o tipo de rigorque os modernistas querem transmitir aos seus alunos e sua ausencia ea maior crıtica que eles fazem aos outros sistemas de ensino.

Finalmente, as distorcoes das proprias ideias modernistas em maosinexperientes levaram a atual situacao do ensino da Matematica Mo-derna no Brasil, onde se da enfase as trivialidades de manejar conjuntos,insiste-se em nuances linguısticas irrelevantes, e estimula-se a mediocri-dade atraves de exercıcios rebuscados sobre o conjunto vazio. Mesmo

Page 35: Dados Internacionais de Cataloga¸cao da Publicac¸ao(CIP) · 2018. 12. 13. · Resolu¸c˜ao Comentada das Provas da XXVI OMEG Valdivino Vargas Junior, Rogerio de Queiroz Chaves,

Revista da Olimpıada de Matematica do Estado de Goias 30

que fosse valida a posicao modernista (e acreditamos que ela e inade-quada e obsoleta), a sua aplicacao no Brasil resultou, com raras excecoes,em um caos completo. E chegado o momento de revermos estas posicoese estruturarmos um ensino mais compatıvel com a Matematica atual emais adequado a realidade brasileira.

Isto nao significa, entretanto, que advoguemos uma volta ao ensinotradicional da Matematica. Uma renovacao nos currıculos e na maneirade ensinar e absolutamente necessaria. Em ummundo cheio de computa-dores, voos espaciais e modelos estatısticos, e indispensavel que o ensinoda Matematica reflita esta nova realidade e ajude a compreende-la.

Esta mudanca, entretanto, deve ser feita tranquilamente, sem eufo-rias exageradas ou promessas irrealizaveis. E essencial que a mudancaobedeca a dos princıpios basicos. Primeiro, que o material a ser ensinadodeva ser util na compreensao do mundo que nos cerca. Segundo, queele possa ser transmitido efetivamente dentro das condicoes brasileiras.Pouco nos adianta copiar currıculos americanos, belgas ou franceses, seo corpo de professores de que dispomos nao permite aplica-los. As mu-dancas devem ser projetados de maneira lenta e progressiva, de modo apermitir aos professores uma adaptacao contınua as novas condicoes.

A tıtulo de exemplo, mencionaremos alguns topicos que provavel-mente podem ser incluıdos no ensino do 1.o grau. Um dos que repre-sentam uma mudanca nos currıculos tradicionais e a Algebra Linear emdimensao de sistemas de coordenadas, vetores e retas no plano, siste-mas de equacoes lineares, matrizes 2× 2 e programacao linear. E claroque estes topicos nao devem ser apresentados secamente em uma ordemlogica. Eles devem ser motivados por problemas e aplicados sempre quepossıvel. A programacao linear pode dar origem a, ou ser motivadapor, pequenos projetos de pesquisas realizados por equipes. A distri-buicao dos topicos ao longo do programa deve obedecer principalmenteas exigencias de motivacao e oportunidade.

A ideia de linearidade e basica na Matematica atual. A introducaoimplıcita desta ideia no ensino do primeiro grau e util e, se possıvel, deveser feita.

Um outro exemplo que demanda modificacoes nos currıculos tradicio-nais e o estudo cuidadoso (porem nao formalmente rigoroso) dos numerosnaturais, inteiros e racionais. A passagem dos numeros naturais para os

Page 36: Dados Internacionais de Cataloga¸cao da Publicac¸ao(CIP) · 2018. 12. 13. · Resolu¸c˜ao Comentada das Provas da XXVI OMEG Valdivino Vargas Junior, Rogerio de Queiroz Chaves,

Revista da Olimpıada de Matematica do Estado de Goias 31

numeros inteiros exige para a sua compreensao o estudo das proprie-dades formais das operacoes com os numeros naturais (comutatividade,associatividade e distributividade). Em verdade, a unica explicacao na-tural para o fato que (−1)(−1) = 1 e que queremos manter nos numerosinteiros as propriedades formais das operacoes com os numeros naturais.Neste caso, justifica plenamente o estudo explıcito destas propriedades.De uma maneira geral a manutencao das propriedades operatorias e umguia indispensavel a construcao de todo o campo numerico, e isto reforcaa importancia do estudo explıcito das propriedades mencionadas.

Outros exemplos incluem a trigonometria do triangulo retangulo,cuja multiplicidade de aplicacoes a torna um instrumento ideal nestenıvel, e a linguagem da teoria dos conjuntos. Dado o seu carater trivial,esta linguagem deve ser introduzida quando necessaria, e sem nenhumaenfase particular.

Como exemplo final, gostarıamos de mencionar a Geometria Euclide-ana. Acreditamos que a Geometria basica de retas, triangulos, polıgonose cırculos deva ser ensinada de maneira sintetica, isto e, atraves de fi-guras e construcoes geometricas. O conteudo intuitivo da GeometriaEuclideana torna este topico um dos mais ricos do currıculo do primeirograu. Nao e razoavel que meras objecoes de rigor formal sejam motivosde subtrair aos alunos as aplicacoes e os desafios a imaginacao que saoproporcionados pelos metodos geometricos. De acordo com o gosto dosalunos, podem se introduzir metodos analıticos auxiliares. Entretantoos metodos geometricos nao devem ser abandonados jamais.

Para concluir estas consideracoes, gostaria de mais uma vez enfati-zar o carater aplicado e computacional que deve ter o ensino da Ma-tematica no primeiro grau. E atraves deste contacto com a realidademais imediata que o aluno aprende a sentir o vigor da Matematica epassa a compreender o papel que ela desempenhou na criacao do mundoque ele ve ao redor de si. Esta compreensao das forcas e limitacoes daMatematica e essencial para que ele possa julgar coisas simples da suaexistencia, tais como o valor do computador ou dos resultados de umapesquisa de opiniao.

Instituto de Matematica Pura e AplicadaRio de Janeiro -Brasil

Page 37: Dados Internacionais de Cataloga¸cao da Publicac¸ao(CIP) · 2018. 12. 13. · Resolu¸c˜ao Comentada das Provas da XXVI OMEG Valdivino Vargas Junior, Rogerio de Queiroz Chaves,

Revista da Olimpıada de Matematica do Estado de Goias 32

Manfredo Perdigao do Carmo (15/08/1928-30/04/2018), Graduou-se em Engenhariapela Universidade Federal de Pernambuco(1951) e doutorado em Matematica pela Uni-versidade da California, Berkeley (1963). FoiProfessor Visitante na Universidade Federaldo Ceara, na Universidade de Brasılia e naUniversidade da California, Berkeley. Pes-quisador Emerito do IMPA. Recebeu a Or-dem Nacional do Merito Cientıfico em 1995.Sua area de atuacao foi a Geometria Dife-rencial onde formou 27 pesquisadores, algunsdos quais sao lıderes nesta area.

Foi membro da Academia Brasileira de Ciencias e da TWAS. Escreveuum livro de Geometria Diferencial publicado em ingles, adotado emvarias universidades no exterior, e traduzido para o alemao, o chines eo russo.

Figura 1.1: Fonte: http://apg.impa.br/lajanela/ Foto de Luis Urbina,fotoografo da Universidad de Murcia, Espanha. Foto providenciada pelaProfessora Leny Cavalcante (esposa do Prof. Manfredo) a quem agra-decemos por tornar possıvel esta homenagem.

Page 38: Dados Internacionais de Cataloga¸cao da Publicac¸ao(CIP) · 2018. 12. 13. · Resolu¸c˜ao Comentada das Provas da XXVI OMEG Valdivino Vargas Junior, Rogerio de Queiroz Chaves,

Revista da Olimpıada - IME - UFG, no- 13, novembro de 2018. 33-40

Jogos e Brincadeiras1

Francisco Bruno de Lima Holanda

Resumo. Neste artigo apresentaremos alguns problemas cujas estrategiassao muito abordadas em olimpıadas de matematica, principalmente nosnıveis iniciais. O primeiro exercıcio trata-se de um jogo nao-estrategico(chamaremos de brincadeira) e os demais sao jogos estrategicos em queas jogadas sao tomadas em turnos.

Problema 1. Quatro garotos jogam tiro ao alvo. Cada um deles ati-rou tres vezes. No alvo abaixo, pode-se ver os lugares atingidos. Apontuacao e 6 para o centro e diminui um ponto para cada nıvel maisdistante. Se os quatro garotos empataram, determine:

⊗ ⊗

⊗⊗

(a) a pontuacao total de cada jogador.

(b) a pontuacao dos tres tiros de cada jogador.

1Artigo preparado para o projeto de extensao Polos Olımpicos de TreinamentoIntensivo (POTI).

Page 39: Dados Internacionais de Cataloga¸cao da Publicac¸ao(CIP) · 2018. 12. 13. · Resolu¸c˜ao Comentada das Provas da XXVI OMEG Valdivino Vargas Junior, Rogerio de Queiroz Chaves,

Revista da Olimpıada de Matematica do Estado de Goias 34

Solucao. A soma de todos os pontos obtidos foi 6+5+4×3+3×3+2× 4 = 40. Como todos empataram, cada um deve ter feito exatamente10 pontos (resposta para o item (a)). Alem disso, e importante perceberque ninguem errou nenhum dos tiros, ja que ha exatamente 12 dardosno alvo. Note que um dos jogadores (digamos A) acertou um dos dardosno centro do alvo, fazendo 6 pontos. Para completar os 10 pontos eledeve ter feito mais 4 pontos. Como e impossıvel fazer apenas 1 ponto,ou dele ter errado, so nos resta a possibilidade dele ter feito 2 pontos nosdois outros tiros. Veja tambem que um dos jogadores fez 5 pontos. Paracompletar os outros cinco pontos, deve ter necessariamente acertado 2e 3 pontos nas suas outras duas jogadas. Um terceiro jogador acertou 4pontos. Para completar os 10 pontos totais, ele pode ter acertado (4 e2) ou (3 e 3). Enquanto que o quarto jogador ficara com as marcacoesrestantes. Dessa forma, podemos resumir as pontuacoes na triplas aseguir: (6, 2, 2) (5, 3, 2) (4, 4, 2) (4, 3, 3)

Problema 2. (OBM 2015 - 3a Fase) Ana e Beatriz possuem muitasmoedas. Elas colocam varias sobre uma mesa e jogam de acordo com asseguintes regras:

i. a primeira a jogar retira no mınimo uma moeda, mas nao todas;

ii. quem jogar a seguir pode retirar no mınimo uma moeda e no maximoo dobro do numero de moedas que o jogador anterior retirou;

iii. ganha quem retirar a ultima moeda.

a) Suponha que elas coloquem 11 moedas sobre a mesa. Se Ana for aprimeira a jogar e retirar duas moedas, mostre como Beatriz podevencer o jogo (nao importando quais sejam as demais jogadas deAna).

b) Agora suponha que elas coloquem 15 moedas sobre a mesa. Mostrecomo a primeira a jogar pode vencer o jogo sempre (nao importandoquais sejam as jogadas da segunda).

Solucao. a) Para Beatriz vencer, deve seguir a seguinte estrategia:

Page 40: Dados Internacionais de Cataloga¸cao da Publicac¸ao(CIP) · 2018. 12. 13. · Resolu¸c˜ao Comentada das Provas da XXVI OMEG Valdivino Vargas Junior, Rogerio de Queiroz Chaves,

Revista da Olimpıada de Matematica do Estado de Goias 35

8

6

7

5

5

4

3

2

1

3

0

0

0

2

1

0

0A

A

B

B

A

A

A

A

B

B

B

B

A

A

B

B

1. Primeiro, ela deve retirar uma moeda, deixando a mesa com oitomoedas.

2. Na vez de Ana, ela podera retirar 1 ou 2 moedas. Deixando a mesacom 7 ou 6 moedas.

3. Neste caso, Beatriz sempre podera deixar a mesa com 5 moedas.

4. Veja que neste ponto, Ana nao podera vencer o jogo. De fato,ela pode retirar no maximo quatro moedas. Caso ela retire duasou mais moedas, Beatriz vence retirando todas as moedas quesobrarem. Caso Ana retire apenas uma moeda, Beatriz deveraretirar mais outra, deixando a mesa com apenas tres moedas.

5. Dessa forma, Ana so podera retirar uma ou duas moedas, e emcada um destes casos, Beatriz podera vencer retirando todas asmoedas que sobrarem.

A figura anterior resume a estrategia que Beatriz deve adotar.

b) Suponha que Beatriz e a primeira a jogar. Vamos mostrar queela sempre pode vencer, independente das jogadas escolhidas por Ana.Basta que ela siga a seguinte estrategia:

Page 41: Dados Internacionais de Cataloga¸cao da Publicac¸ao(CIP) · 2018. 12. 13. · Resolu¸c˜ao Comentada das Provas da XXVI OMEG Valdivino Vargas Junior, Rogerio de Queiroz Chaves,

Revista da Olimpıada de Matematica do Estado de Goias 36

1. O primeiro passo e retirar duas moedas deixando 13 moedas paraAna. Se Ana retirar, em sua jogada, cinco ou mais moedas, Beatrizpodera vencer retirando todas as moedas restantes em sua proximajogada.

2. Caso Ana retire duas, tres ou quatro moedas, Beatriz deve retiraruma quantidade suficiente de moedas para entregar oito moedaspara Ana. Se, em sua jogada, Ana retirar uma ou duas moedas,Beatriz deve prosseguir com a mesma estrategia mencionada noitem (a) deste exercıcio. Caso Ana retire tres ou mais moedas,Beatriz vence retirando as moedas restantes.

3. Se Ana retirar uma moeda na rodada anterior, deixando 12 moedasna mesa, Beatriz deve retirar apenas uma moeda, deixando 11 namesa. Dessa forma, Ana so podera retirar uma ou duas moedas.Em qualquer caso, Beatriz podera retirar uma quantidade sufici-ente de moedas para deixar a mesa com exatamente oito moedas.Reduzindo a estrategia do ponto anterior.

A figura a seguir resume a estrategia que Beatriz deve adotar.

13

9

10

11

12 11

8

8

8

10

9

8

8

AA

B

B

B

B

A

A

B

B

Alguns tipos de jogos possuem certas configuracoes que sempre levamum jogador a vitoria. Essas configuracoes sao chamadas de posicoes

Page 42: Dados Internacionais de Cataloga¸cao da Publicac¸ao(CIP) · 2018. 12. 13. · Resolu¸c˜ao Comentada das Provas da XXVI OMEG Valdivino Vargas Junior, Rogerio de Queiroz Chaves,

Revista da Olimpıada de Matematica do Estado de Goias 37

vencedoras. O proximo exemplo e um jogo bastante simples em queessa estrategia aparece facilmente.

Problema 3. Na primeira casa de um tabuleiro 1×13 esta uma moeda.Tiago e Maria movem a moeda alternadamente. Em cada turno e permi-tido avancar 1, 2, 3, 4 ou 5 casas. Quem colocar a moeda na ultima casae o vencedor. Se Maria comecar jogando, ela pode ter certeza da vitoria?

Solucao. Como em muitos problemas de olimpıada, vamos analisaralguns casos pequenos. Vamos supor que em vez de 13 casas o tabuleirotivesse apenas quatro. Neste caso, fica facil ver que quem comeca ganha,basta avancar tres casas.

O mesmo iria ocorrer se o tabuleiro tivesse 2, 3, 4, 5 ou 6 casas.Porem, em um tabuleiro 7× 1 o primeiro jogador perde. Veja que aposa primeira jogada a moeda estara em uma das casas 2, 3, 4, 5 ou 6. E jasabemos que essas casas levam o jogador a vitoria.

Desse modo, vamos dizer que 7 e uma posicao perdedora e 6, 5, 4, 3e 2 sao posicoes vencedoras. Assim, se um o jogador estiver em umadas casas 8, 9, 10, 11 ou 12, ele pode garantir a vitoria movendo a moedapara a casa 7, deixando o seu adversario em uma posicao perdedora.Com isso, podemos afirmar que as posicoes 8, 9, 10, 11 e 12 tambem saoposicoes vencedoras.

Resta analisar a 13a casa. Observe que a partir desta casa podemosmover a moeda apenas para uma das casas 8, 9, 10, 11 ou 12 que saovencedoras. Daı, quem comecar perde pelo simples fato de iniciar emuma posicao perdedora.

A grande dificuldade para a maioria dos alunos e descobrir quais saoas posicoes vencedoras de um jogo. Para evitar esse tipo de problema,tenha sempre em mente as seguintes definicoes:

Page 43: Dados Internacionais de Cataloga¸cao da Publicac¸ao(CIP) · 2018. 12. 13. · Resolu¸c˜ao Comentada das Provas da XXVI OMEG Valdivino Vargas Junior, Rogerio de Queiroz Chaves,

Revista da Olimpıada de Matematica do Estado de Goias 38

(a) Posicao vencedora: A partir dela, podemos escolher ummovimento e repassar uma posicao perdedora para o adversario.

(b) Posicao perdedora: A partir dela, e impossıvel escolher ummovimento e repassar uma posicao perdedora para o adversario.Ou seja, nao importa o movimento escolhido, o adversario irareceber uma posicao vencedora.

E como fazer para descobrir quais sao as posicoes vencedoras e per-dedoras? A melhor maneira de se fazer isto e analisando o final do jogoe aplicar as definicoes acima. Vamos praticar um pouco resolvendo oproximo problema.

Problema 4. Em um tabuleiro 8× 8, uma torre esta na casa a1. Doisjogadores movem a torre com objetivo de colocar a torre na casa h8.Sabendo que a torre pode mover-se apenas para cima ou para direita(quantas casas o jogador desejar) e que nao pode-se passar a vez, deter-mine qual jogador tem a estrategia vencedora.

Solucao. Primeiramente note que todas as casas da ultima linhae da ultima coluna (exceto a h8) sao vencedoras pois, a partir delaspodemos escolher um movimento que nos leve a vitoria. Com, isso acasa g7 se torna perdedora pois, a partir dela qualquer movimento levao outro jogador a uma posicao vencedora (veja a figura 1).

Figura 1

a b c d e f g h

12345678

Figura 2 Figura 3

Agora, como g7 e perdedora, as demais casas da setima linha e dasetima coluna sao vencedoras. Mais ainda, a casa f6 tambem deve ser

Page 44: Dados Internacionais de Cataloga¸cao da Publicac¸ao(CIP) · 2018. 12. 13. · Resolu¸c˜ao Comentada das Provas da XXVI OMEG Valdivino Vargas Junior, Rogerio de Queiroz Chaves,

Revista da Olimpıada de Matematica do Estado de Goias 39

perdedora (figura 2). Continuando de maneira analoga, obtemos que acasa a1 e perdedora (figura 3). Logo, quem comecar, perde.

Problemas Propostos

Problema 5. Sob uma mesa estao 38 moedas. Alberto e Bianca jogamem turnos. Em cada turno e permitido retirar 1, 2, 3 ou 4 moedas. Quemretirar as ultimas moedas e o vencedor. Se Bianca comecar jogando, elapode ter certeza da vitoria?

Problema 6. Uma pilha de 500 pedras e dada. Dois jogadores jogam oseguinte jogo: Em cada turno, o jogador pode retirar 1, 2, 4, 8, ... (qual-quer potencia de 2) pedras da pilha. O jogador que nao puder maisjogar perde.

Problema 7. Em uma caixa existem 300 bolinhas. Cada jogador poderetirar nao mais do que a metade das bolinhas que estao na caixa. Ojogador que nao puder mais jogar perde.

Problema 8. Sobre uma mesa existem duas pilhas (uma com 7 e outracom 15 pedras). Em um jogo cada jogador pode, em sua vez, retirarqualquer quantidade de pedras de apenas uma pilha ou a mesma quan-tidade de ambas as pilhas. Quem nao puder mais jogar perde. Quempossui a estrategia vencedora?

Bibliografia Recomendada

Muitos dos exercıcios propostos nesta aula foram retirados dos livros:

1. Dmitri Fomin, Sergey Genkin, Ilia V. Itenberg. MathematicalCircles: Russian Experience (Mathematical World, Vol.7). American Mathematical Society 1996.

2. Dmitry Fomin, Alexey Kirichenko. Leningrad MathematicalOlympiads 1987-1991 (Contests in Mathematics Series.Vol. 1). MathPro Press 1994.

Page 45: Dados Internacionais de Cataloga¸cao da Publicac¸ao(CIP) · 2018. 12. 13. · Resolu¸c˜ao Comentada das Provas da XXVI OMEG Valdivino Vargas Junior, Rogerio de Queiroz Chaves,

Revista da Olimpıada de Matematica do Estado de Goias 40

Outra fonte de problemas sao as paginas da Olimpıada Brasileira de Ma-tematica (www.obm.org.br) e da Olimpıada Brasileira de Matematicade Escolas Publicas (www.obmep.org.br).

Dicas e Solucoes

5. Este jogo e muito similar ao jogo proposto no problema 3 destematerial. Veja que os multiplos de 5 sao posicoes perdedoras.Neste caso, Bianca vence retirando tres moedas. E toda vez queCarlos retirar x moedas, Bianca deve retirar 5− x moedas em seuturno.

6. Pense nos multiplos de 3 (sao as posicoes perdedoras). Lembre-seque nenhuma potencia de 2 e multiplo de 3.

7. Pense nas potencias de 2.

8. Novamente, use a ideia do tabuleiro que foi usada para resolver oproblema 4. Mais precisamente, pense no jogo como um tabuleiro8 × 16 com uma peca no canto inferior esquerdo. Retirar umaquantidade x de moedas da primeira pilha correspondera movera peca x casas para cima na vertical. Retirar uma quantidade yde moedas da segunda pilha correspondera mover a peca y casaspara a direita na horizontal. Retirar uma quantidade z de moedasdas duas pilhas correspondera mover a peca z casas para cima e zcasas para a direita na diagonal. Dessa forma, o objetivo do jogopassa a ser chegar na casa do canto superior direito do tabuleiro.

Autor: Francisco Bruno de Lima Holanda

Endereco: Professor Adjunto da Faculdade de Administracao,Contabilidade e Economia FACE-UFG,e-mail: [email protected]

Page 46: Dados Internacionais de Cataloga¸cao da Publicac¸ao(CIP) · 2018. 12. 13. · Resolu¸c˜ao Comentada das Provas da XXVI OMEG Valdivino Vargas Junior, Rogerio de Queiroz Chaves,

Revista da Olimpıada - IME - UFG, no- 13, novembro de 2018. 41-57

Potencia de um Ponto e Eixo Radical

Emiliano Augusto Chagas

Resumo. No final do ensino fundamental, os alunos entram emcontato com alguns conhecimentos geometricos especıficos sobre circun-ferencias. Particularmente, os conceitos de angulo inscrito e central emuma circunferencia, permitem o desenvolvimento de muitas questoes deolimpıada de matematica e tambem servem de alicerce para outros re-sultados interessantes. Em seguida, os alunos tomam contato com oTeorema das Cordas e o Teorema das Secantes, que relacionam medidasde segmentos sob o efeito de uma circunferencia. Nesse artigo vamosexplorar o conceito de Potencia de Ponto, que e essencia desses ultimosdois teoremas mencionados, e seus desdobramentos. Durante o desen-volvimento da teoria alguns problemas de olimpıada serao resolvidos,no final voce pode encontrar uma lista com diversos problemas, cujaresolucao envolve o conteudo presente nesse artigo. Bons estudos!

Potencia de um Ponto em Relacao a uma Circunferencia

Definicao: Seja C uma circunferencia de centro O e raio R. Se umponto P esta a uma distancia d de O, definimos a potencia do ponto Pem relacao a circunferencia C por:

Pot(PC) = d2 −R2.

Pela definicao apresentada, se P e exterior a C, sua potencia e umnumero positivo; se P pertence a C, sua potencia e zero e, se P einterior a C, sua potencia e negativa. Passemos entao a investigar aspropriedades desse conceito.

Teorema 1. Sao dados uma circunferencia C e um ponto P . Se umareta passa por P e corta C nos pontos A e B, entao o produto PA ·PBe constante.

Page 47: Dados Internacionais de Cataloga¸cao da Publicac¸ao(CIP) · 2018. 12. 13. · Resolu¸c˜ao Comentada das Provas da XXVI OMEG Valdivino Vargas Junior, Rogerio de Queiroz Chaves,

Revista da Olimpıada de Matematica do Estado de Goias 42

Demonstracao. Seja O o centro de C e R o seu raio. Seja P um pontoqualquer no plano, chamamos PO = d. Consideremos uma secante PABe o ponto M , ponto medio de AB. Facamos MA = MB = m e, notamostambem que OM e perpendicular a AB, uma vez que a mediatriz de ABpassa pelo centro O da circunferencia:

Temos tres casos a considerar, uma vez que o ponto P e arbitrario,em relacao a posicao relativa de P com a circunferencia C.

(a) Se P e exterior a C.

R

OP

A

B

M

PA ·PB = (PM −m)(PM +m) = PM2−m2. Como o trianguloPMO e retangulo, podemos utilizar o teorema de Pitagoras e es-crever PM2 = PO2 − MO2. Substituindo na equacao anteriortemos PO2 − MO2 − m2 = PO2 − (MO2 + m2). Pelo Teoremade Pitagoras, novamente, veja que MO2 + m2 = R2. Fazendo asubstituicao, temos PO2 − (MO2 +m2) = PO2 −R2 = d2 −R2.Entao: PA · PB = Pot(PC)

(b) Se P e interior a C.

RO

A

B

MP

PA · PB = (m − PM)(m + PM) = m2 − PM2. Novamente,como o triangulo PMO e retangulo, podemos utilizar o teoremade Pitagoras e escrever PM2 = PO2 − MO2. Substituindo naequacao anterior temos m2 − (PO2 −MO2) = m2 +MO2 −PO2.

Page 48: Dados Internacionais de Cataloga¸cao da Publicac¸ao(CIP) · 2018. 12. 13. · Resolu¸c˜ao Comentada das Provas da XXVI OMEG Valdivino Vargas Junior, Rogerio de Queiroz Chaves,

Revista da Olimpıada de Matematica do Estado de Goias 43

Pelo Teorema de Pitagoras, novamente, veja que MO2+m2 = R2.Fazendo essa substituicao, temos m2+MO2−PO2 = R2−PO2 =R2 − d2.Entao: PA · PB = −Pot(PC)

(c) Se P pertence a circunferencia. Nesse caso temos que A ou Bcoincide com P , um dos dois segmentos tem comprimento zero ea potencia de P e tambem igual a zero.

Esse desenvolvimento ate agora nos evidencia o Teorema das Cordase o Teorema das Secantes, uma vez que foi mostrado que o produtoPA ·PB e constante. Mais do que constante, agora sabemos exatamentequanto vale esse produto, e agora podemos utilizar esse conhecimentoao nosso favor.

Na realidade, o fato que o produto PA·PB e constante para qualquersecante passando por P e conhecido desde a antiguidade, mas o termo“potencia” foi utilizado pela primeira vez por Jacob Steiner (1796-1863),matematico suıco que deu uma enorme contribuicao ao desenvolvimentoda Geometria.

Veja que temos um caso particular relevante aqui. Se P e exteriora circunferencia e se PT e tangente em T , decorre da definicao (e doteorema de Pitagoras) que Pot(PC) = PT 2.

R

OP

T

Em relacao ao apresentado ate agora, e possıvel categorizar os problemasque envolvem Potencia de Ponto em duas categorias. A primeira e aquelaem que apenas utilizamos o fato de que a potencia de um ponto emrelacao a uma circunferencia e constante, como nos casos do Teoremadas Cordas ou das Secantes. A segunda utiliza a potencia de um pontopara relacionar a distancia desse ponto ao centro da circunferencia com oraio dessa (d2 −R2). Vamos trabalhar com esses dois tipos de problemaagora.

Page 49: Dados Internacionais de Cataloga¸cao da Publicac¸ao(CIP) · 2018. 12. 13. · Resolu¸c˜ao Comentada das Provas da XXVI OMEG Valdivino Vargas Junior, Rogerio de Queiroz Chaves,

Revista da Olimpıada de Matematica do Estado de Goias 44

Problema 9. (Banco Jr Balkan) Prove que nao existe um trianguloem que a bissetriz interna de um angulo em A corte o incırculo (cir-cunferencia inscrita) de triangulo em X e Y (X entre A e Y ) de modoque a ceviana AD produzida pela bissetriz interna seja dividida em tressegmentos congruentes: AX, XY e Y D.

Solucao. Esse problema e interessante, pois em geral as questoesde geometria nos pedem para provar determinados angulos, segmentos,concorrencias ou coisas do tipo, e nao provar a impossibilidade de al-guma configuracao. Apos esbocar alguns triangulos com seus respectivosincırculos, voce comeca a perceber que de fato nao ha como a cevianaAD ser dividida em tres segmentos congruentes como se pede. Considerea figura a seguir.

B

A

CD

F

G

X

Y

Vamos comecar a encontrar algumas relacoes entre as medidas uti-lizando potencia de ponto. Vamos chamar o incırculo de Γ, alem disso,seja AX = XY = Y D = k. Os pontos F e G sao pontos de tangenciaentre os lados do triangulo e o incırculo.

Pot(AΓ) = AF 2 = AX · AY = k · 2k = 2k2 e

Pot(DΓ) = GD2 = DY ·DX = k · 2k = 2k2.

Desse modo temos que

AF 2 = 2k2 = GD2 ⇔ AF = GD.

Pot(BΓ) = BF 2 = BG2 ⇔ BF = BG.

Perceba entao que, utilizando o resultado anterior, temos que

BA = BF +AF = BG+BD = BD,

Page 50: Dados Internacionais de Cataloga¸cao da Publicac¸ao(CIP) · 2018. 12. 13. · Resolu¸c˜ao Comentada das Provas da XXVI OMEG Valdivino Vargas Junior, Rogerio de Queiroz Chaves,

Revista da Olimpıada de Matematica do Estado de Goias 45

em outras palavras, o triangulo ABD e isosceles.Portanto, ∠BDA = ∠BAD, e como AD e bissetriz interna do angulo

no vertice A ainda temos que ∠BAD = ∠DAC, ou seja, ∠BDA =∠DAC. E isso e um absurdo, uma vez que isso so ocorreria se os ladosBD e AC fossem paralelos. Logo, essa configuracao nao existe.

Problema 10. (IMO 2009) Seja ABC um triangulo de circuncentro O.Os pontos P e Q sao pontos internos dos lados CA e AB, respectiva-mente. Sejam K, L e M os pontos medios dos segmentos BP , CQ ePQ respectivamente, e seja Γ a circunferencia que passa por K, L e M .Suponha que a reta PQ e tangente ao cırculo Γ. Prove que OP = OQ.

Solucao. Existem outras maneiras de resolver esse problema, masvamos tentar entender o motivo de usar potencia de ponto nesse pro-blema. Como precisamos provar que OP = OQ, sendo O o centro deuma circunferencia, isso seria equivalente a provar

R2 −OP 2 = R2 −OQ2 = Pot(P ) = Pot(Q).

O problema possui pontos medios, que ajuda muito na transposicaode angulos alem de proporcionar diversos triangulos semelhantes. Alemdisso temos uma circunferencia tangente a um segmento, novamenteisso facilita na transposicao de angulos. Vejamos como fica a figura doproblema:

A

B C

P

Q

K

L

M

Vamos comecar pela circunferencia. Como Γ e tangente a PQ, temosque ∠QMK = ∠MLK pois enxergam o mesmo arco. Veja que M eK sao pontos medios, portanto MK ‖ AB e desse modo ∠QMK =∠AQM . Essas duas ultimas equacoes envolvendo angulos nos levam a

Page 51: Dados Internacionais de Cataloga¸cao da Publicac¸ao(CIP) · 2018. 12. 13. · Resolu¸c˜ao Comentada das Provas da XXVI OMEG Valdivino Vargas Junior, Rogerio de Queiroz Chaves,

Revista da Olimpıada de Matematica do Estado de Goias 46

∠AQP = ∠MLK. Analogamente, utilizando M e L pontos medios,chegamos em ∠MKL = ∠APQ.

Uma marcacao de angulo que poderia ter sido utilizada, e que e maisintuitiva que essas duas, e a seguinte: M e K sao pontos medios acarretaem MK ‖ AB, M e L pontos medios acarreta em ML ‖ AC, e dessemodo ∠KML = ∠QAP .

Olhando os angulos nos triangulos QAP e KML temos entao que

∆QAP ∼ ∆KML, portantoAQ

ML=

AP

MK. Vamos usar agora que MK

e ML sao bases medias de QB e PC, respectivamente:

AQ

2ML=

AP

2MK⇔ AQ

PC=

AP

QB⇔ QA ·QB = PA · PC.

Mas veja que essa ultima igualdade significa dizer que P e Q pos-suem a mesma potencia em relacao a circunferencia Γ, como tınhamosprevisto. Finalmente,

QA ·QB = PA · PC ⇔ −Pot(QΓ) = −Pot(PΓ) ⇔R2 −OQ2 = R2 −OP 2 ⇔ OP = OQ.

A Distancia do Incentro ao Circuncentro

E interessante o fato de que, em qualquer triangulo, o raio da circun-ferencia circunscrita nao e menor que o dobro do raio da circunferenciainscrita. Esse resultado, aparentemente difıcil de demonstrar, decorreimediatamente da formula que da a distancia entre o incentro e o cir-cuncentro de um triangulo, como veremos a seguir.

Sejam I e O o incentro e o circuncentro do triangulo ABC, respec-tivamente. Alem disso, tomemos os raios das circunferencias inscrita ecircunscrita a ABC, e chamamos de r e R respectivamente.

A primeira pergunta que voce deve se fazer e a seguinte: por que aPotencia de Ponto pode nos dizer qual e a distancia entre o incentro e ocircuncentro? A resposta nao e difıcil, pela definicao de Potencia de umPonto P em relacao a uma circunferencia C, no caso de P interno a C,temos

PA · PD = −Pot(PC) = −(PO2 −R2) = R2 − PO2,

onde AD e uma corda de C.

Page 52: Dados Internacionais de Cataloga¸cao da Publicac¸ao(CIP) · 2018. 12. 13. · Resolu¸c˜ao Comentada das Provas da XXVI OMEG Valdivino Vargas Junior, Rogerio de Queiroz Chaves,

Revista da Olimpıada de Matematica do Estado de Goias 47

Se tomarmos a potencia do ponto I em relacao a circunferencia,teremos entao IA · ID = R2 − IO2. Veja que apareceu um termo IO,que e exatamente a distancia desejada.

Considere a figura do triangulo ABC a seguir. AD e bissetriz doangulo A e, consequentemente, D e o ponto medio do arco BC. Ainda,DE e um diametro. Seja F o pe da altura relativa ao lado AC, por-tanto IF e perpendicular a AC e IB esta contido na bissetriz do angulo∠ABC. Sejam α a medida dos angulos ∠DAB, ∠DAC, ∠DBC e∠DCB e β a medida dos angulos ∠IBC e ∠IBA.

B C

A

O

D

E

I

F

M

Observe que:

∠IBD = ∠IBC + ∠DBC = α+ β e

∠BID = ∠IBA+ ∠DAB = α+ β.

Portanto DB = DI e analogamente DC = DI.Observando que os triangulos IFA e EBD sao retangulos e ∠IAF =

∠DAB = ∠DEB, concluımos que sao semelhantes. Assim,

IF

IA=

BD

DE⇔ 2Rr = IA×BD

Mas por potencia, sabemos que IA · BD = IA · ID = R2 − IO2,donde obtemos IO2 = R2 − 2Rr. Ou seja:

IO =√

R2 − 2Rr =√

R(R− 2r).

Essa e a expressao que da a distancia entre o incentro e o circun-centro de um triangulo em funcao dos raios das circunferencias inscritae circunscrita, obtida pela primeira vez por Euler. Quanto ao fato que

Page 53: Dados Internacionais de Cataloga¸cao da Publicac¸ao(CIP) · 2018. 12. 13. · Resolu¸c˜ao Comentada das Provas da XXVI OMEG Valdivino Vargas Junior, Rogerio de Queiroz Chaves,

Revista da Olimpıada de Matematica do Estado de Goias 48

citamos no inıcio desta secao, observe que devemos ter R2 − 2Rr ≥ 0,o que implica R ≥ 2r. Repare ainda que a igualdade so ocorre quandoo incentro coincidir com o circuncentro, ou seja, quando o triangulo forequilatero.

Particularmente e difıcil encontrar problemas em olimpıada que en-volvem essa informacao, o resultado por si so ja e incrıvel o suficientepara nao precisar aparecer em nenhuma olimpıada. Mas vejamos agoraum raro problema que utiliza essa ideia na resolucao.

Problema 11. (Balcanica 1986) Uma reta passando pelo incentro I dotriangulo ABC intersecta a circunferencia circunscrita Γ1 do ∆ABC, decentro O e raio R, nos pontos F e G, e o incırculo Γ2, de centro I e raior, nos pontos D e E, com D entre I e F . Prove que DF · EG ≥ r2.Quando ha igualdade?

Solucao. Vamos considerar a figura a seguir:

A

B C

I

F D

E

G

Veja que DF e EG se relacionam com o incentro I, uma vez queesses tres pontos estao alinhados e ainda por cima, podemos escreverDF e EG em termos de IF e IG. Desse modo, a potencia do ponto Iem relacao a circunferencia maior aparece.

DF ·EG = (IF −DI)(IG− EI) = (FI − r)(GI − r)

= FI ·GI − (IF + IG)r + r2 = −Pot(IΓ1)− FG.r + r2.

Pela relacao de Euler, da distancia do incentro ao circuncentro, temosque OI2 = R2 − 2Rr, e assim temos Pot(IΓ1

) = OI2 −R2 = −2Rr.Queremos mostrar entao que

2Rr − FG · r + r2 ≥ r2 ⇔ 2Rr ≥ FG · r ⇔ 2R ≥ FG,

que e verdade, pois FG e corda de Γ1. A igualdade ocorre se e so se FGe diametro de Γ1.

Page 54: Dados Internacionais de Cataloga¸cao da Publicac¸ao(CIP) · 2018. 12. 13. · Resolu¸c˜ao Comentada das Provas da XXVI OMEG Valdivino Vargas Junior, Rogerio de Queiroz Chaves,

Revista da Olimpıada de Matematica do Estado de Goias 49

Eixo Radical e Centro Radical

Qual sera o lugar geometrico dos pontos que possuem mesma potenciaem relacao a duas circunferencias dadas?

Consideremos duas circunferencias (nao concentricas) de centros A eB, e raios R e r respectivamente. Seja AB = 2d e seja M o ponto mediode AB. Tomemos um ponto P que tenha mesma potencia em relacao asduas circunferencias e seja H a projecao de P sobre AB como mostra afigura a seguir. Seja ainda PM = m e α a medida do angulo ∠PMH.Se P tem a mesma potencia em relacao as duas circunferencias, entao:

PA2 −R2 = PB2 − r2 ⇔ PA2 − PB2 = R2 − r2.

A B

P

M H

α

A lei dos cossenos nos triangulos PMA e PMB fornece:

PA2 = m2 + d2 + 2md cosα e PB2 = m2 + d2 − 2md cosα.

Subtraindo as duas equacoes, temos

PA2 − PB2 = 4md cosα ⇔R2 − r2 = 4md cosα = 2AB ·MH ⇔

MH =R2 − r2

2AB.

PortantoMH e constante, e dessa forma, o pontoH e fixo sobre AB. As-sim, o lugar geometrico de P e a reta perpendicular a AB passando porH. Logo, o conjunto dos pontos que possuem mesma potencia em relacaoa duas circunferencias e uma reta perpendicular a reta que contem osdois centros e e chamada de eixo radical das duas circunferencias. E in-teressante observar que, se as duas circunferencias forem secantes, o eixo

Page 55: Dados Internacionais de Cataloga¸cao da Publicac¸ao(CIP) · 2018. 12. 13. · Resolu¸c˜ao Comentada das Provas da XXVI OMEG Valdivino Vargas Junior, Rogerio de Queiroz Chaves,

Revista da Olimpıada de Matematica do Estado de Goias 50

radical contem os pontos de interseccao e, se as circunferencias foremtangentes, o eixo radical contem o ponto de tangencia. Logo, quandoduas circunferencias sao secantes ou tangentes, a construcao do eixoradical e imediata. Mas como obter o eixo radical quando duas circun-ferencias nao tem ponto comum? Uma solucao possıvel e a seguinte:

Dadas duas circunferencias C1 e C2, de centros A e B respectiva-mente, tracemos uma circunferencia C de centro W , qualquer, que sejasecante as duas primeiras. Na figura, a circunferencia C cortou a circun-ferencia C1 em D e E, e cortou a circunferencia C2 em F e G. A retaDE e o eixo radical entre C1 e C e a reta FG e eixo radical entre C2 e C.Assim, o ponto P , interseccao de DE com FG tem a mesma potenciaem relacao as tres circunferencias e e chamado de centro radical de C1,C2 e C. Logo, P pertence ao eixo radical das circunferencias C1 e C2 e,para desenha-lo, basta conduzir por P uma reta perpendicular a AB.

A B

W

D

EF

G

P

Problema 12. (USAMO 2009) Sejam Γ1 e Γ2 circunferencias que seintersectam em X e Y . Seja r a reta que passa pelo centro de Γ1 eintersecta Γ2 em P e Q, e seja s a reta que passa pelo centro de Γ2 eintersecta Γ1 em R e S. Prove que, se P , Q, R e S estiverem em umamesma circunferencia, entao o centro dessa circunferencia esta na retaXY .

Solucao. Considere a figura a seguir:

Page 56: Dados Internacionais de Cataloga¸cao da Publicac¸ao(CIP) · 2018. 12. 13. · Resolu¸c˜ao Comentada das Provas da XXVI OMEG Valdivino Vargas Junior, Rogerio de Queiroz Chaves,

Revista da Olimpıada de Matematica do Estado de Goias 51

O1

O2

X

Y

Q

P

SR

O3

Seja Γ3 o circuncırculo (circunferencia circunscrita) de PQRS. Vejaque O1 esta na reta r, que passa por PQ. Sabemos que se duas circun-ferencias se intersectam, o eixo radical dessas circunferencias passa poresses pontos de interseccao. Sendo assim, O1 esta no eixo radical de Γ2

e Γ3. Analogamente, O2 esta no eixo radical de Γ1 e Γ3. Seja ri o raioda circunferencia Γi, desse modo temos:

Pot(O1Γ2) = Pot(O1Γ3

) ⇔ O1O22 − r2

2 = O1O32 − r3

2

Pot(O2Γ1) = Pot(O1Γ3

) ⇔ O2O12 − r1

2 = O2O32 − r3

2.

Fazendo a subtracao dessas equacoes temos

O1O32 − r1

2 = O2O32 − r2

2 ⇔ Pot(O3Γ1) = Pot(O3Γ2

),

portanto O3 esta no eixo radical de Γ1 e Γ2, desse modo X, Y e O3 saocolineares.

Uma definicao equivalente para o eixo radical, no caso em que as duascircunferencias Γ1 e Γ2 possuem no maximo um ponto de interseccao, e aque o eixo radical e o lugar geometrico dos pontos P tais que as tangentesde P as duas circunferencias Γ1 e Γ2 tem o mesmo comprimento.

Problema 13. Se a distancia entre os centros de duas circunferenciasΓ1 e Γ2 e maior do que a soma de seus raios, as circunferencias temquatro tangentes em comum. Prove que os pontos medios desses quatrosegmentos sao colineares.

Solucao. A condicao do problema garante que a posicao relativade Γ1 e Γ2 e a da figura.

Page 57: Dados Internacionais de Cataloga¸cao da Publicac¸ao(CIP) · 2018. 12. 13. · Resolu¸c˜ao Comentada das Provas da XXVI OMEG Valdivino Vargas Junior, Rogerio de Queiroz Chaves,

Revista da Olimpıada de Matematica do Estado de Goias 52

A

BM

Vamos mostrar que cada um dos pontos medios esta no eixo radicalde Γ1 e Γ2. De fato, se M e o medio de alguma dessas tangentes AB,entao MA = MB . Pela Definicao Equivalente de Eixo Radical, temosque M esta no eixo radical de Γ1 e Γ2. Como o mesmo vale para osoutros pontos medios, concluımos que os quatro pontos sao colineares.

Problemas Para Esquentar

Problema 14. Sejam Γ1 e Γ2 duas circunferencias que se intersectam.Seja a tangente comum a Γ1 e Γ2 tocar Γ1 em A e Γ2 em B. Mostreque a corda comum de Γ1 e Γ2, quando prolongada, bissecta o segmentoAB, ou seja, divide o segento AB em duas partes congruentes.

Problema 15. (Rioplatense 2001) Seja AD a altura relativa ao ladoBC do triangulo acutangulo ABC. M e N sao os pontos medios doslados AB e AC, respectivamente. Seja E o segundo ponto de interseccaodas circunferencias circunscritas aos triangulos BDM e CDN . Mostreque a reta DE passa pelo ponto medio de MN .

Problema 16. (Jr Balkan 2012) Sejam as circunferencias k1 e k2 e seuspontos de interseccao A e B. Seja tambem t a tangente comum a k1 ek2 nos pontos M e N respectivamente. Se t e perpendicular a AM eMN = 2AM , encontre ∠NMB.

Problema 17. Seja C um ponto num semicırculo de diametro AB eseja D o ponto medio do arco AC. Seja E a projecao de D na reta BCe F a interseccao da reta AE com o semicırculo. Prove que BF bissectao segmento DE.

Problema 18. Sejam A, B e C tres pontos em uma circunferencia Γcom AB = BC. Seja D o encontro das tangentes em A e B. Seja

Page 58: Dados Internacionais de Cataloga¸cao da Publicac¸ao(CIP) · 2018. 12. 13. · Resolu¸c˜ao Comentada das Provas da XXVI OMEG Valdivino Vargas Junior, Rogerio de Queiroz Chaves,

Revista da Olimpıada de Matematica do Estado de Goias 53

E a outra interseccao de DC com Γ. Prove que a reta AE bissecta osegmento BD.

Problema 19. (Banco Cone Sul 2002) Seja ABCD um quadrilateroinscritıvel e E a intersecao das diagonais AC e BD. Se F e um pontoqualquer e as circunferencias Γ1 e Γ2 circunscritas a FAC e a FBD seintersectam novamente em G, mostre que E, F , G sao colineares.

Problemas Avancados

Problema 20. Seja ABC um triangulo agudo. Considere a reta porB perpendicular a AC intersectar a circunferencia de diametro AC nospontos P e Q, e considere a reta por C perpendicular a AB intersectara circunferencia de diametro AB nos pontos R e S. Prove que P , Q, Re S sao concıclicos (PQRS e um quadrilatero inscritıvel).

Problema 21. Seja ABC um triangulo agudo de ortocentro H. Ospontos M e N sao tomados nos lados AB e AC respectivamente. Ascircunferencias de diametro BN e CM intersectam-se em P e Q. Proveque P , Q e H sao colineares.

Problema 22. Sejam ABC um triangulo e D e E pontos nos lados ABe AC, respectivamente, tal que DE e paralelo a BC. Seja P um pontoqualquer no interior do triangulo ADE, e sejam F e G as interseccoes deDE com as retas BP e CP , respectivamente. Seja Q a outra interseccaodos circuncırculos dos triangulos PDG e PFE. Prove que os pontos A,P e Q sao colineares.

Problema 23. (Teste de Selecao Ibero 2002) Seja ABCD um qua-drilatero inscrito em uma circunferencia Γ1, P o ponto de intersecao dasdiagonais AC e BD e M o ponto medio de CD. A circunferencia Γ2 quepassa por P e e tangente a CD em M corta BD e AC nos pontos Q eR, respectivamente. Seja S o ponto do segmento BD tal que BS = DQ.A paralela a AB por S corta AC em T . Prove que AT = CR.

Problema 24. (IMO 1995) Sejam A, B, C, D pontos distintos em umareta, nesta ordem. As circunferencias Γ1 e Γ2 de diametros AC e BDse intersectam em X e Y . O e um ponto arbitrario da reta XY , naosituado em AD. CO intersecta Γ1 novamente em M , e BO intersectaΓ2 novamente em N . Prove que AM , DN e XY sao concorrentes.

Page 59: Dados Internacionais de Cataloga¸cao da Publicac¸ao(CIP) · 2018. 12. 13. · Resolu¸c˜ao Comentada das Provas da XXVI OMEG Valdivino Vargas Junior, Rogerio de Queiroz Chaves,

Revista da Olimpıada de Matematica do Estado de Goias 54

Problema 25. (Ibero 1999) Um triangulo acutangulo ABC esta inscritonuma circunferencia de centro O. As alturas do triangulo sao AD, BEe CF . A reta EF intersecta a circunferencia em P e Q.

(a) Prove que AO e perpendicular a PQ.

(b) Se M e o ponto medio de BC, prove que AP 2 = 2ADOM .

Problema 26. (Ibero 1998) A circunferencia inscrita no triangulo ABCe tangente aos lados BC, CA e AB nos pontos D, E e F , respectiva-mente. AD corta a circunferencia em um segundo ponto Q. Demons-tre que a reta EQ passa pelo ponto medio de AF se, e somente se,AC = BC.

Problema 27. (Ibero 2004) Consideramos no plano uma circunferenciade centro O e raio r e um ponto A exterior a ela. Seja M um ponto dacircunferencia e N o ponto diametralmente oposto a M . Achar o lugargeometrico dos centros das circunferencias que passam por A, M e Nao variar M .

Problema 28. (Ibero 1999) Dadas duas circunferencias M eN , dizemosque M bissecta N se a corda comum e um diametro de N . Considereduas circunferencias nao concentricas C1 e C2.

(a) Prove que existem infinitas circunferencias B tais que B bissectaC1 e B bissecta C2.

(b) Determine o lugar geometrico dos centros das circunferencias B.

Problema 29. (Jr Balkan 2005) Seja ABC um triangulo acutanguloem uma circunferencia k. A tangente a circunferencia por A encontraa reta BC no ponto P . Seja M o ponto medio do segmento AP e Ro segundo ponto de interseccao da circunferencia k com a reta BM . Areta PR encontra novamente a circunferencia k no ponto S diferente deR. Prove que as retas AP e CS sao paralelas.

Problema 30. (IMO 2008) SejaH o ortocentro de um triangulo acutan-gulo ABC. A circunferencia ΓA centrada no ponto medio de BC epassando por H intersecta o lado BC nos pontos A1 e A2. Similarmente,defina os pontos B1, B2, C1 e C2. Prove que os seis pontos A1, A2, B1,B2, C1 e C2 sao concıclicos.

Page 60: Dados Internacionais de Cataloga¸cao da Publicac¸ao(CIP) · 2018. 12. 13. · Resolu¸c˜ao Comentada das Provas da XXVI OMEG Valdivino Vargas Junior, Rogerio de Queiroz Chaves,

Revista da Olimpıada de Matematica do Estado de Goias 55

Problema 31. (USAMO 1997) Seja ABC um triangulo. Construatriangulos isosceles BCD, CAE e ABF externamente a ABC de ba-ses BC, CA e AB, respectivamente. Prove que as retas que passam porA, B, C e sao perpendiculares a EF , FD e DE, respectivamente, saoconcorrentes.

Problema 32. (IMO 2000) Dois cırculos, Γ1 e Γ2, intersectam em M eN . Seja l a tangente comum a Γ1 e Γ2 tal que M e mais perto de l doque N . A interseccao de l com Γ1 e A e com Γ2 e B. Seja a reta por Mparalela a l encontrar Γ1 novamente, em C e Γ2, novamente em D. Asretas CA e BD se encontram em E; as retas AN e CD se encontramem P ; as retas BN e CD se encontram em Q. Mostre que EP = EQ.

Problema 33. (USAMO 1998) Sejam C1 e C2 cırculos concentricos,com C2 no interior de C1. Seja A o ponto em C1 e B um ponto em C2 talque AB e tangente a C2. Seja C o segundo ponto de interseccao de AB eC1, e seja D o ponto medio de AB. Uma reta passando por A intersectaC2 em E e F de modo que a mediatriz de DE e CF intersectam em um

ponto M de AB. Encontre a razaoAM

MC.

Resolucao dos Problemas sem Referencia

6) Seja M a interseccao do prolongamento da corda comum com AB.Como M esta no eixo radical de Γ1 e Γ2, tem a mesma potencia comrespeito as duas circunferencias, portanto MA2 = MB2 ⇔ MA = MB.

9) Seja Γ o cırculo de diametro AB e Γ1 o cırculo de diametro BE.Como ∠AFB = 90, Γ1 passa por F . E como ∠DEB = 90, Γ1 e tan-gente a DE. Entao, pelo problema 1, a corda comum BF , de Γ e Γ1

bissecta a tangente comum DE.

10) Seja Γ1 o circuncırculo de ADE. Pelo problema 1 e sufici-ente mostrar que Γ1 e tangente a DB. De fato, temos que ∠ADB =180−2∠ABD = ∠ABC = ∠AEC, o que implica que Γ1 e tangente aD.

11) O eixo radical de Γ1 e Γ2 e a reta FG. Entao E e um ponto deFG ⇔ Pot(EΓ1

) = Pot(EΓ2). Mas ABCD inscritıvel implica Pot(EΓ1

) =−AE ·EC = −BE ·ED = Pot(EΓ2

), e portanto E, F e G sao colineares.

Page 61: Dados Internacionais de Cataloga¸cao da Publicac¸ao(CIP) · 2018. 12. 13. · Resolu¸c˜ao Comentada das Provas da XXVI OMEG Valdivino Vargas Junior, Rogerio de Queiroz Chaves,

Revista da Olimpıada de Matematica do Estado de Goias 56

12) Seja D o pe da perpendicular de A ate BC, e seja H o ortocentrode ABC. Como ∠ADB = 90, a circunferencia de diametro AB passapor D, entao HS · HR = HA · HD por potencia de ponto. De modosimilar, a circunferencia de diametro AC passa por D tambem, entaoHP ·HQ = HA ·HD. Portanto HP ·HQ = HR ·HS, e portanto pelavolta da potencia de ponto, P , Q, R e S sao concıclicos.

13) Nos queremos mostrar queH esta no eixo radical de dois cırculos,entao e suficiente mostrar queH tem a mesma potencia com respeito aosdois cırculos. Sejam BE e CF as duas alturas de ABC. Como ∠BEN =90 , E esta na circunferencia de diametro BN . Portanto a potencia deH com respeito a circunferencia de diametro BN e HBHE. Analoga-mente, a potencia de H com respeito a circunferencia de diametro CMe HCHF . Como ∠BEC = ∠BFC = 90, B, C, E, F sao concıclicos,portanto HB · HE = HC · HF por potencia de ponto. Segue que Htem a mesma potencia com relacao as duas circunferencias de diametroAB e BC.

14) Seja M a interseccao do circuncırculo de DPG com a reta AB,e seja N a interseccao do circuncırculo de EPF com a reta AC. As-suma uma configuracao onde M e N estao nos lados AB e AC, res-pectivamente (o argumento para o outro caso e similar). Temos que∠ABC = ∠ADG = 180 − ∠BDG = 180 − ∠MPC, entao BMPCe cıclico. Similarmente BPNC e cıclico tambem. Entao BCNPMe cıclico, portanto ∠ANM = ∠ABC = ∠ADE, e M , N , D, E saoconcıclicos. Pela potencia de ponto, AD ·AM = AE ·AD. Portanto, Atem a mesma potencia com respeito aos circuncırculos de DPG e EPF ,e portanto A esta na reta PQ, o eixo radical.

Bibliografia

[1] Lima, Y. G., Potencia de Ponto, Eixo Radical, Centro Radical eAplicacoes.https://www.obm.org.br/content/uploads/2017/01/eixos-2.pdf

[2] Wagner, E., Potencia de um Ponto em Relacao a uma Circun-ferencia. Revista do Professor de Matematica, Vol. 45, p. 29 - 34,SBM, 2001.

Page 62: Dados Internacionais de Cataloga¸cao da Publicac¸ao(CIP) · 2018. 12. 13. · Resolu¸c˜ao Comentada das Provas da XXVI OMEG Valdivino Vargas Junior, Rogerio de Queiroz Chaves,

Revista da Olimpıada de Matematica do Estado de Goias 57

[3] Branzei, D., Serdean, I., Serdean, V. Junior Balkan,Mathematical Olympiads. Plus, 2003.

[4] Fauring, P. et al., Olimpıadas Iberoamericanas de Matematicas,1996 - 2006. Red Olımpica, 2007.

Autor: Emiliano Augusto Chagas

Endereco: Instituto Federal de Sao Paulo, IFSPRua Pedro Vicente, 625 - Caninde -Sao Paulo - SP - Brasil - Cep: 01109-010e-mail: [email protected]

Page 63: Dados Internacionais de Cataloga¸cao da Publicac¸ao(CIP) · 2018. 12. 13. · Resolu¸c˜ao Comentada das Provas da XXVI OMEG Valdivino Vargas Junior, Rogerio de Queiroz Chaves,

Revista da Olimpıada - IME - UFG, no- 13, 2018

Objetivo e Polıtica Editorial

A Revista da Olimpıada tem como objetivo ser um veıculo dedifusao, principalmente, das Olimpıadas de Matematica do Estado deGoias, promovidas pelo IME/UFG.

A Revista tambem esta aberta a contribuicoes de pequenas materias,subordinados a boa qualidade. O material submetido para a publicacaodevera ser de interesse do Ensino Fundamental e Medio, estar bem re-digido, em estilo claro, sem aridez, de forma que desperte o interesse doleitor.

Submissao e Aceite

Toda materia submetida para publicacao deve ser enviada ao ComiteEditorial. Materias redigidas em TEX ou LATEX podem ser submetidaspor e-mail: [email protected]. Se existirem ilustracoes no trabalho sub-metido, estas devem ser encaminhadas, juntamente com o trabalho, eprecisam estar em condicoes de serem reproduzidas, sem retoques. Alemdisso, copias dos desenhos e ilustracoes devem ser afixadas em espacosapropriados do texto, exibindo, dessa maneira, como devera ficar a apre-sentacao final do trabalho.

As referencias bibliograficas devem ser colocadas no final do texto,em ordem alfabetica, segundo as normas da ABNT.

As materias submetidas para publicacao serao analisadas pelos edi-tores que poderao solicitar pareceres ad hoc e o autor recebera a respostasobre sua materia num prazo maximo de 120 dias.

Os autores que tiverem os trabalhos aceitos deverao transferir seusdireitos autorais para o Instituto de Matematica e Estatıstica da UFG.

Page 64: Dados Internacionais de Cataloga¸cao da Publicac¸ao(CIP) · 2018. 12. 13. · Resolu¸c˜ao Comentada das Provas da XXVI OMEG Valdivino Vargas Junior, Rogerio de Queiroz Chaves,